Этого треда уже нет.
Это копия, сохраненная 16 ноября 2016 года.

Скачать тред: только с превью, с превью и прикрепленными файлами.
Второй вариант может долго скачиваться. Файлы будут только в живых или недавно утонувших тредах. Подробнее

Если вам полезен архив М.Двача, пожертвуйте на оплату сервера.
46 Кб, 300x363
10 Кб, 234x300
28 Кб, 300x300
95 Кб, 1015x1381
МАТЕМАТИКА ДЛЯ НАЧИНАЮЩИХ, ТРЕД 12, ОТПУСКНОЙ! #377293 В конец треда | Веб
ДЛЯ САМЫХ МАЛЕНЬКИХ:

Общие курсы
М. И. Сканави: "Элементарная математика".

Алгебра
И. М. Гельфанд, А. Шень: “Алгебра”. Весь курс школьной алгебры по 9 класс.
С. Б. Гашков: “Современная элементарная алгебра”.

Геометрия
А. Д. Александров, А. Л. Вернер, В. И. Рыжик: “Геометрия”. Учебник для 10-11 классов. Базовый и углубленный уровни.
Я. П. Понарин: “Элементарная геометрия” в двух томах. Первый том - это планиметрия, а второй том - это стереометрия.
А. Ю. Калинин, Д. А. Терешин: “Геометрия”, 10-11 классы. Годный учебник.

Тригонометрия
И. М. Гельфанд, С.М. Львовский, А. Л. Тоом: “Тригонометрия”. Название говорит само за себя. Много геометрических и физических интерпретаций + комплексные числа, как бонус.

Начала анализа
Б. М. Давидович: “Математический анализ в 57 школе“.

БАЗОВЫЕ КУРСЫ ДЛЯ СТУДЕНТОВ:

Общая алгебра
Э. Б. Винберг: “Курс алгебры”. Пожалуй, лучший из известных учебников, соперничать с которым может разве что "Введение в алгебру" Кострикина.
А. И. Кострикин: “Введение в алгебру“. Пожалуй, лучший из известных учебников, соперничать с которым может разве что "Курс алгебры" Винберга.
М. Атья, И. Макдональд : "Введение в коммутативную алгебру".
А. Л. Городенцев: "Алгебра. Учебник для студентов-математиков". Вырос из лекций НМУ. Читать параллельно с Винбергом (Винберга читать в первую очередь).
И.Р. Шафаревич: “Основные понятия алгебры“. Замечательный обзор вообще того, что такое алгебра, как она выглядит и какое место она занимает в математике. Примеры, приложения и прочая конкретика.
E. Connell: Elements of Abstract and Linear Algebra". Хорошая первая книга по алгебре, да и математике вообще.
P. Grillet: "Abstract algebra". Очень лаконичный и понятный учебник. Надо знать элементарную теорию чисел, про индукцию, про множества и функции. Линейной алгебры нету.
J. Rotman: "Advanced modern algebra". Ротман сильно разжевывает. Задачи слишком простые для уровня учебника. Линейная алгебра есть.
M. Artin: "Algebra". Американский Винберг. Группы Ли, упор на геометрию. Задачи неудачные.
I. Herstein: “Topics in Algebra“. Прекрасные задачи, отбор материала очень устарел, почти что Ван дер Варден.
P. Aluffi: "Algebra, Chapter 0". Если ты в состоянии ее осилить, бери и забывай про остальные книжки из списка. Линейная алгебра есть.

Линейная алгебра
В. А. Ильин, Э. Г. Позняк: “Линейная алгебра“. Один из классических и самых популярных курсов линейной алгебры.
Д. В. Беклемишев: “Курс аналитической геометрии и линейной алгебры“.
И. М. Гельфанд: "Лекции по линейной алгебре". Не даётся определение определителя.
А. И. Кострикин, Ю. И. Манин: "Линейная алгебра и геометрия". Затрагивается темы геометрий и связей с квантовой механикой. Не даётся определение определителя.
S. Axler: "Linear algebra done right". Подход без определителей (почти). Одна из самых популярных книг за рубежом.
S. Treil: "Linear algebra done wrong". Не такая популярная, как Axler, но тоже хвалят, да. Определители есть.
G. Shilov: "Linear Algebra". Определитель появляется на первой странице.
K. Hoffman, R. Kunze: "Linear Algebra". Классика за рубежом.
P. Halmos: "Finite-Dimensional Vector Spaces". Тоже классика.
P. Peterson: "Linear Algebra". Не особо знаком, но выглядит аккуратно. Что-то вроде Акслера.
S. Roman: "Advanced Linear Algebra". Хороший учебник по линалу. Но нужно знать элементарные свойства матриц и определителей.

Математический анализ
T. Tao: “Real analysis“. Один из самых популярных курсов математического анализа на английском языке.
C. Pugh: "Real Mathematical analysis". Более простая версия Рудина с картинками. Норм книга, но не самая лёгкая.
У. Рудин: "Основы математического анализа".
В. А. Зорич: "Математический анализ". Первый том посвящен классическому анализу. Много примеров, много материала, в том числе даются в начале основы матлогики и теории множеств, а также функций между ними.
Р. Курант: "Курс дифференциального и интегрального исчисления". Идеален с точки зрения первого знакомства с теорией, но имеет достаточно сложные упражнения.
Г. М. Фихтенгольц: "Курс дифференциального и интегрального исчисления". Хорош как повторительный курс.
С. М. Львовский: "Лекции по математическому анализу". Записки лекций из НМУ. Нужно знать основы калькулюса.
Г. Г. Харди, Д. Е. Литтлвуд, Г. Пойа: "Неравенства".
Н. Н. Лебедев: "Специальные функции и их приложения".
Г. П. Толстов: “Ряды Фурье“.

Дифференциальные уравнения
С. Фарлоу: “Уравнения с частными производными для научных работников и инженеров“.

Вариационное исчисление
И. М. Гельфанд, С. В. Фомин: " Вариационное исчисление".

Топология
V. Runde: "A taste of topology". Неплохая книга по метрическим пространствам и общей топологии, затрагивает фундаментальную группу.
J. Strom: "Modern classical homotopy theory".
T. Dieck: "Algebraic topology".
M. Crossley: "Essential Topology". Пререквизит для изучения алгебраической топологии. Не затрагивает тему метрических пространств.

КУРСЫ ДЛЯ ПРОДВИНУТЫХ МАТЕМАТИКОВ

Математический анализ
А. И. Маркушевич: "Теория аналитических функций".
S. Ramanan: "Global calculus".
H. Amann, J. Echer: "Analysis".
W. Fidcher, I. Lieb: "A Course in Complex Analysis: From Basic Results to Advanced Topics".

Дифференциальные уравнения
В. И. Арнольд: “Обыкновенные дифференциальные уравнения”. Книга для уверенных в себе математиков. Диффеоморфизмы, фазовые потоки, гладкие многообразия. Слава Гермесу Трисмегисту!

Теория категорий
С. Маклейн: "Категории для работающего математика".
Р. Голдблатт: "Топосы. Категорный анализ логики".

Дифференциальная Геометрия
К. Номидзу: "Основы дифференциальной геометрии".
J. Lee: "Manifolds and DIfferential Geometry".
L. Nicolaescu: "Lectures on the Geometry".
P. Michor "Topics in Differential Geometry".

Алгебраическая геометрия
Д. Мамфорд: "Красная книга о многообразиях и схемах".
В. В. Острик, М. А. Цфасман: “Алгебраическая геометрия и теория чисел: рациональные и эллиптические кривые”.
В. И. Арнольд: “Вещественная алгебраическая геометрия”.
Ю. И. Манин: Введение в теорию схем и квантовые группы“.
R. Vakil: "Foundations of algebraic geometry".
S. Bosch: "Algebraic Geometry and Commutative Algebra".
U. Gotz, T. "Wedhorn: Algebraic Geometry".
E. Harris: "The Geometry of Schemes".

Топология
А. Хэтчер: "Алгебраическая топология".
J. Munkres: "Topology". Книга - жесткий учебник по теоретико-множественной топологии. Много ненужного для других областей математики.

ИНТЕРЕСНОЕ:

Цикл “Manga guide to...“. Популярное изложение различных областей математики (и не только), оформленное в виде манги. Увы, без фансервиса.
Н. А. Вавилов: “Конкретная теория групп I: основные понятия“. И вообще все остальные книги (и лекции!) Вавилова.
П. С. Александров: “Введение в теорию групп“. Просто о сложном. Несколько вольный язык изложения, местами затрудняющий восприятие.
В. Б. Алексеев: “Теорема Абеля в задачах и решениях”.
Р. Курант, Г. Роббинс: “Что такое математика?”. Очень интересная книга, в двух словах не описать. Но вас захватит, надолго.
Н. Я. Виленкин: "Рассказы о множествах". Теория множеств для широкого круга читателей.
М. М. Постников: “Теорема Ферма. Введение в теорию алгебраических чисел”.
Н. Стинрод: “Первые понятия топологии“.
А. Я. Хинчин: “Три жемчужины теории чисел“.
О. Я. Виро, О. А. Иванов, Н. Ю. Нецветаев, В. М. Харламов: “Элементарная топология”.
Я. П. Понарин: “Алгебра комплексных чисел в геометрических задачах”.
А. А. Заславский: “Геометрические преобразования”.
В. Акопян, А. А. Заславский: “Геометрические свойства кривых второго порядка”.
В. И. Арнольд: “Геометрия комплексных чисел, кватернионов и спинов”.
В. В. Прасолов: “Геометрия Лобачевского”.
Д. В. Аносов: “Дифференциальные уравнения: то решаем, то рисуем”.
В. В. Прасолов: “Наглядная топология”.
Д. В. Аносов: “От Ньютона к Кеплеру”.
М. Клайн: “Математика. Поиск истины“.
Д. Пойа: “Математическое открытие“.
Л. Кэрролл: “Логическая игра“.
Д. Пойа: “Как решать задачу“.
О. Я. Виро, Д. Б. Фукс: "Введение в теорию гомотопий. Гомологии и когомологии".
A. Ostermann, G. Wanner: "Geometry by its history".
T. Sundstrom: "Mathematical reasoning writing and proof". В книге объясняется что такое математическое доказательство, математический факт и каким образом их можно придумывать. Начала теории множеств.
D. Dummit R. Foote: “Abstract Algebra“. Много примеров, задач, но страшно скучный учебник, его нужно держать как справочник.

ПОЛЕЗНЫЕ РЕСУРСЫ:

Библиотка "Квант": math.ru/lib/ser/bmkvant
Высшая математика просто и доступно, по 2 курс включительно: mathprofi.net
Необъятная онлайн библиотека: gen.lib.rus.ec
>>377628>>378725
150 Кб, 800x600
#2 #377297
первонах! сфоткал переходящий приз маттреда. выкуплю его у петровича пожалуй.
>>377298
#3 #377298
>>377297
Какой у него багровый гребень. И бородка тоже.
>>377300
188 Кб, 600x800
#4 #377300
>>377298
мне теперь везде эти петухи мерещатся. вот еще из коллекции. это кафе аист напротив саранского политеха мы там с пацанами N определние обсуждали. заскучал, смотрю на стену - а там гениталии петуха. в смысле сверху гребень а снизу яйца и уретра. может спьяни померещилось. а вы аноны что на этой картине видите?
>>377302>>377314
#5 #377302
>>377300
Сук, ну ты уже задал психологическую установку. Теперь всем, кто ее прочтет придется усилие делать, чтобы то же самое не увидеть.
>>377304
#6 #377304
>>377302
Ну да, ну а помнишь всякие картинки на обман зрения - типа кого видишь, молодуху или старуху. Сперва увидишь одну, а потом другую, и можешь переключаться. А здесь кроме гениталий петуха я ничего не вижу, как ни старался!
>>377305
#7 #377305
>>377304
Если бы я был психоаналитиком, то, скорее всего, сказал бы, что тебе стоит поменьше с сидеть на дваче.
#8 #377310
Опять ОП список авторства товарищей Говнова и Жабаебалагадюкова.
Когда то мне перестанет быть лень и я создам тред математики для не математиков.
#9 #377314
>>377300
Это символ Франции, le coq.
35 Кб, 1542x168
#10 #377315
Суть чистой математики
>>377335
#11 #377318
Зачем вы вообще математикой такого уровня занимаетесь? С неё же нет проку. "Для себя" ответ не катит.
#12 #377319
>>377318
Да тут никто и не занимается. Студентишки, освоившие азы АГ максимум.
>>377332>>377334
#13 #377332
>>377319
А ты?
#14 #377334
>>377319
N-петух, почему не отвечаешь на вопрос? Тебя приз ждет на пике, если правильно ответишь. У тебя же хирш и публикации!
#15 #377335
>>377315
из какой-нибудь гедельэшербах цитата?
>>377339
#16 #377336
>>377318
Зачем вы вообще существуете? С вас же нет проку. "Для себя" ответ не катит.
>>377338
#17 #377338
>>377336
некоторые вообще танчики клеят и чуствуют себя прекрасно. но у них же все свободное пространтсво говном забито. а тут блокнот и карандаш, и лаптоп. и весь lebensraum.
#18 #377339
>>377335
Да. Чет мегагодная книжка.
#19 #377345
>>377318
Как нет проку, можно тянкам в уши заливать, они текут, когда парень умнее их, еще можно аутировать в голове безо всякого интернета, сплошные плюсы.
>>377347>>377351
#20 #377347
>>377345

> можно тянкам в уши заливать, они текут, когда парень умнее их


И что именно ты тянкам рассказываешь? Типа как в видосах numberphile? Мне бы западло было такую хуйню рассказывать.
#21 #377351
>>377345
Чтобы "парень умнее их" достаточно посмотреть с десяток фильмов и почитать с десяток книг, плюс анализ всего этого. Математическими концепциями, даже "интересными" можно доказать, что ты скучный мудак.
>>377358>>377363
#22 #377358
>>377351

> Математическими концепциями, даже "интересными" можно доказать, что ты скучный мудак.


А если тянку научить правильному счету (ноль, целковый, полушка, четвертушка, осьмушка, пудовичек, медячек, серебрячек, золотнячек), она потечет?
>>377377
#23 #377362
>>377318
1) интересненько
2) зарабатывать бабосы
>>377405
#24 #377363
>>377351
Напомнили - меня как-то недавно затащили на концерт на открытом воздухе, на лужаечке епта. Я чуть не охуел - столько народу давно не видел вместе! И громко, что пиздец. И приперлись как-то сильно заранее, хотя я заметил, что необязательно прямо лезь к сцене. Можно неподалеку лежать на травке, не так громко и все равно все видно хотя хули там смотреть. Я тем временем вытащил блокнотик и стал прикидывать за сколько до начала лучше всего приходить, чтобы от скуки не охуеть и видно было нормально. Я пытался объяснить, что по идее sweet spot должен быть на пересечении двух графиков - функции заполняемости (размер толпы от времени) и ф-ции "кайфа" в завимости от удаленности от сцены. Функция заполняемости мне кажется какой-то long tailed sigmoid (только не гуглите в images умоляю, страшные картинки выползают), вернее не Long tailed но с перекосом не знаю как назвать, основная масса народа приходит "внезапно" и сравнительно незадолго до начала, а потом асимптотически приближается к full house. А функция кайфа еще интереснее - оказывается практически похую где находиться, кроме самых задворков, где нихуя не видно. Так что пересечение приходится на примерно 30 мин до начала. А пускать начинают часа за 3, это охуеть, и есть ебланы которые к открытию приходят.

Короче я попытался это все рассказывать, но меня никто не слушал и смотрели как на мудака. Хм, а мне кажется это интересно.
>>377367
#25 #377367
>>377363
Когда меня люди спрашивают, чем занимаешься, я обычно отвечаю "ничем", потому что если ответить "математика", то есть небольшой риск, что спросят чем конкретно, и тут придётся придумывать неимоверно скучную научно-популярную историю, чтобы как-то оправдаться, типа "представь себе бублик, есть значит такие пространства..."
>>377375>>377385
#26 #377373
Ребят, не нашел отдельного треда, обращаюсь сюда. Подскажите хорошей литературы по дискретной математике.
>>377379>>377414
#27 #377375
>>377367
лол, жызненно
#28 #377377
>>377358
Нет, потечет Юрий Степанович.
#29 #377379
>>377373
Ну ты и спросил. Половина математики под определение "дискретной" попадает.
>>377412
#30 #377385
>>377367
Верно, потому что ты занимаешься ненужным дерьмом. Так что ответ "ничем" даже более точен.
>>377386
#31 #377386
>>377385
в чем критерий "нужности"? вот нужник действительно нужен. остальное спорно.
>>377388>>377389
#32 #377388
>>377386
Не можешь сходу пояснить за свою деятельность -- не нужен.
>>377403>>377411
#33 #377389
>>377386
Не уверен, что нужен, -- не нужен.
>>377403
#34 #377403
>>377388
>>377389

>АЗАЗАЗА ВАШЕ УВЛИЧИНЕЯ НИНУЖНА МОЯ НУЖДНА ЗАТРОЛИЛВАЛИВАЛ ЛАЛОККК!!!!

>>377406
#35 #377405
>>377362

>математика


>зарабатывать бабосы


ПОТЕРПЕЛ СОКРУШИТЕЛЬНОЕ ПОРАЖЕНИЕ
>>377407
#36 #377406
>>377403
И это все что ты можешь сказать?!
>>377408
#37 #377407
>>377405
Перельман пять лет отработал в США на маленьких должностях. Заработал столько денег, что до сих пор ими свою семью кормит.
>>377424
#38 #377408
>>377406
Говорить о нужности чего-либо бессмысленно, так как нужность обуславливается субъективным отношением человека к объекту дискуссии.

Ясно епта?!
>>377425
#39 #377411
>>377388
Глупости. Сейчас во всех областях человеческих знаний очень высокая степень специализации. Часто даже компетентному человеку нужно прослушать целый доклад, чтоб хотя бы в общих чертах понять, чем другой занимается.
>>377428
#40 #377412
>>377379
Логические высказывания, графы, автоматы - вот это все.
мимо другой анон
30 Кб, 318x404
#41 #377414
>>377373
Пикрелейтед. Но читать его следует строго на английском, в русском переводе есть убийственно безобразные опечатки.
>>377449
#42 #377415
Как думать про циклические группы? Всякие там аддитивные группы Zn и roots of unity мне кажутся совершенно искусственными, хочется из жизни какую-нибудь группу, которую я буду представлять.
>>377418>>377423
#43 #377418
>>377415
1. Сложение беззнаковых чисел в программировании. Z/2^2Z, Z/2^8Z, Z/2^16Z, Z/2^32Z.
2. Целые числа. Z
>>377419
#44 #377419
>>377418

> Сложение беззнаковых чисел в программировании. Z/2^2Z, Z/2^8Z, Z/2^16Z, Z/2^32Z.


Слишком простые группы. У них даже подгрупп нет кроме тривиальной.

> 2. Целые числа. Z


Это не из жизни.
>>377421>>377422
#45 #377421
>>377419

> кроме тривиальной


и самой себя
#46 #377422
>>377419

>Слишком простые группы.


Все циклические группы слишком простые

>У них даже подгрупп нет кроме тривиальной.


Есть. Уже у Z/2^8Z 7 нетривиальных подгрупп.
#47 #377423
>>377415
Минуты Z/60Z, часы Z/24Z.
#48 #377424
>>377407
Тогда почему он в швецию укатил? Там же все ДОРАХА
>>377427
#49 #377425
>>377408
Давай сравним нужность тупой пизды с айфончиком и бабы нюры из деревни.
#50 #377427
>>377424
Умный человек, понимает к чему всё идёт. Но это тема для другой доски.
>>377429
#51 #377428
>>377411

>О, вася привет, чем занимаешься?


>Да вот гамалогии с гаматопияями считаю.


>ЩИТО?


>Ну короче считаю просто.


>А что такое гамалогии с гаматопиями?


>... Ой бля иди нахуй



>О вася привет, что делаешь?


>на скорителе работаю


>ЩИТО?


>Ну атомы по кругу гоняю под сабатон


>ясн)

>>377431
#52 #377429
>>377427
Ты понимаешь что это просто гнилые отмазки!?
Тот же уровень ответа на вопрос "зачем ты качаешься?" будет "вон ероха выиграл на мистере олимпия и уже десять лет кормит семью"
#53 #377431
>>377428

>О, вася привет, чем занимаешься?


>Да вот гамалогии с гаматопияями считаю.


>ЩИТО?


>Ну короче считаю просто.


>А что такое гамалогии с гаматопиями?


>... Ой бля иди нахуй


Какая-то нереальная ситуация. Обычный человек больше всего на свете боится че-то новое узнать, он просто ненавидит знания. Когда он чего-то не понимает, он просто ждет пока тема разговора сменится или сам пытается перевести разговор в другое русло. Сильно сомневаюсь, что он будет спрашивать, что такое гомологии. Тянка может спросить для поддержания разговора, но ей будет абсолютно похуй на твой ответ, потому что вы с ней не общаетесь, а просто выполняете ритуал.
>>377433
#54 #377433
>>377431

>Обычный человек больше всего на свете боится че-то новое узнать


Как там диван? Не сильно жмет?
>>377437>>377440
#55 #377437
>>377433
Ну хз, может это специфика Мухосранска, в котором я живу, но все мои знакомые обсуждают вещи типа как они на велике покатались, новый расфорсенный пиндосский фильм про супергероев (suicide squad или как его), футбол и прочую даунскую хуйню. Я как-то на вписке объяснил, почему Q счетно, а R несчетно. Люди послушали, поняли (это любой школьник поймет, если на пальцах рассказывать), а потом мне саркастически так говорят: "Охуенно. Вот я приду на собес, у меня спросят, что я умею, а я скажу, что могу доказать, что вещественных чисел больше чем натуральных". То есть, у 99% людей отношение к интеллектуальной деятельности чисто утилитарное: если что-то требует умственных затрат, но не приносит денег - этим занимаются только долбоебы и аутисты.
#56 #377438
>>377437

>Пришло в голову рассказывать диагональный аргумент на вписке


Тебе когда-нибудь говорили, что ты долбоёб?
>>377439
#57 #377439
>>377438
Обычно просто нажираемся, происходит какой-то трэшак, а на следующий день смешно. Но меня заебало каждый раз одно и то же делать, решил разнообразие внести.

> Тебе когда-нибудь говорили, что ты долбоёб?


Only too often.
#58 #377440
>>377433
Я не он, но отвечу.
Он имеет в виду, что большинство людей поглощены собой и им похуй на тебя и твои интересы. 1 из 10 задаст тебе пару осмысленных вопросов и выслушает ответы. 1 из 1000 пойдёт потом и почитает что-то там сам по этой теме будь то математика или разведение скота. Где-то 3 из 10 просто сядут тебе на уши и будут нон-стоп рассказывать как у них дела и про свою работу. А с остальными можно говорить только об общих для всех людей интересах: бабло, еда, отношения etc.

Я могу ебать детей 10 лет и мои друзья не узнают, потому что они никогда не спрашивают чем я вообще занимаюсь и как у меня дела.

Естественно, это мой личнный опыт. Может мне не везёт просто.
>>377441>>377443
#59 #377441
>>377440

> Я могу ебать детей 10 лет и мои друзья не узнают, потому что они никогда не спрашивают чем я вообще занимаюсь и как у меня дела.


К тебе уже выехали.

> А с остальными можно говорить только об общих для всех людей интересах: бабло, еда, отношения etc.


Да, именно это я и хотел сказать.
#60 #377443
>>377440

>мои друзья никогда не спрашивают чем я вообще занимаюсь и как у меня дела.


Заебись у тебя друзья)

Похоже мы тут открыли филиал бэ, но да и похуй
>>377444
#61 #377444
>>377443

> мы тут открыли филиал бэ


Ну так тут все такие аутисты-маргиналы из Мухосрансков, которые не могут общаться в нормальном математическом коммьюнити. Без обид анон, но если бы мне было с кем в жизни обсуждать математику, я бы тут не сидел.
>>377445>>377451
#62 #377445
>>377444
Я бы сидел. Нужно 2-3 норм персонажа, чтобы создать коммунити. Ну и сама идея -- сосака оплот современной математики -- почему-то забавляет.

Да и чисто идеалистически: на каком-нибудь форуме с аватарками начинается ЧСВшное брожение, а тут ононимные посты -- главное идеи, а не люди.
>>377448
#63 #377446
>>377437

>почему Q счетно, а R несчетно


Вангую аргументацию в духе
"Ну кароч вот числа адин джва три есть, да, а есть дроби 0.999999999999......9 нутыпонял. Так вот, таки чисел дохуя, а все остальные посчитать можно"
#64 #377448
>>377445
Сосака - оплот эзотерико-мистической математики.

> Да и чисто идеалистически: на каком-нибудь форуме с аватарками начинается ЧСВшное брожение


На форуме да. А если в жизни общаться, то все збс будет, наверное. Если кто-то будет вести себя как Мунин на dxdy, я ему просто пизды дам.
#65 #377449
>>377414
Спасибо тебе, анон
#66 #377451
>>377444

>не могут общаться в нормальном математическом коммьюнити


Говно без задач. Если не в курсе, советую посмотреть какой-нибудь "буквальная геометрия" (есть на рутрекре), вполне отражает суть.
#67 #377452
>>377451
Проиграл с коментов
https://www.youtube.com/watch?v=cruPpXnS4i4
#68 #377456
>>377437
Это ты долбоёб, а не они, реплику-то их ты так и не понял. Естественно, их твоя школьная хуйня не впечатлила (вангую, они в любом быдлонаучпопном видосике уже это всё слышали). Они очевидно ждали чего-то более крышесносящего, типа вопросов об основаниях математики и обосновании ряда натуральных.
>>377467
#69 #377467
>>377456

> типа вопросов об основаниях математики и обосновании ряда натуральных


Почему ты думаешь, что он выступал перед N-петухами?
>>377473
#70 #377468
>>377451
Так это наполовину их, бездуховное скучное комньюнити.
Не то что наше, с каледиными; геями, наркоманами, художниками, активистами лгтб c матфака и преподами-педофилами из 57.
>>377472
#71 #377472
>>377468
Интересные личности это здорово конечно, вот только общение с ними ровно такое же будет, как в этих коммьюнити. Причем кем бы ты ни был, хоть их родственником.

Типичная ошибка сознания, тащем-то. В смысле думать, что с людьми вроде Вербицких или Михайловых в реале приятно общаться.
>>377475>>377485
#72 #377473
>>377467
Все люди рождаются N-петухами.
#73 #377475
>>377472
С Вербицким приятно, кстати. Очень вежливый и умный собеседник с хорошим голосом. Я так не умею.
#74 #377485
>>377472

>вот только общение с ними ровно такое же будет, как в этих коммьюнити


Что-то я не увидел, а в чём говённость того комьюнити?
#75 #377487
Что по мат. статистике почитать посоветуете?
>>377568
#76 #377492
Умножаю десятичные дроби на целые числа на ханакадеми. Скоро буду с вами наравне.
>>377494
#77 #377493
>>377318
Все что ни делается в этой жизни, делается для себя, маня. Вот ты, сразу видно, мышлением обделен, математикой не занимался
#78 #377494
>>377492
Молодец.

>>377451

> советую посмотреть какой-нибудь "буквальная геометрия" (есть на рутрекре), вполне отражает суть


Пиздец у них там нудная картофанская область. Никогда не понимал, что за аутисты таким занимаются.
>>377495
#79 #377495
>>377494
Благодарю покорно
#80 #377505
>>377318
Чтобы понимать лекции Романа Михайлова и выебываться на дваче. Нет, серьезно.
>>377506
#81 #377506
>>377505
Нуок. А ты понимаешь лекции Ромы Михайлова?
>>377507
#82 #377507
>>377506
Пока еще нет, но думаю что в течение полугода начну понимать. Я сейчас почти изучил пререквизиты к алгебраической топологии.
>>377508
#83 #377508
>>377507
Что именно? Каким путём идёшь к успеху? Давай, делись опытом. Я тоже хочу понимать лекции Ромы Михайлова. Кстати, а нахуя тебе лекции Ромы Михайлова?
>>377513
#84 #377512
В чем разница между разными изданиями книг? Вот я хочу скачать калькулус томаса, а там целых 13 переизданий. В чем суть переизданий?
>>377514>>377515
#85 #377513
>>377508

> Кстати, а нахуя тебе лекции Ромы Михайлова?


Ну я посмотрел видосы с ним, подумал: "вот это чувака штырит, хочу, чтоб меня так же штырило" и скачал учебник по топологии, начал изучать всякую элементарщину, которую по-хорошему надо было в школе пройти: непрерывность, компактность, связность. Почитал че такое quotient topology, понял как "склеивать" всякие хуевины друг с другом, но пока этим почти не занимался. Ну меня вштырило в принципе.

Какие-то элементарнейшие факты про группы, кольца и поля я знаю, поэтому, если собраться с силами и ебаться с книгой Хэтчера каждый день, попутно восполняя пробелы из других областей, можно уже сейчас ее осилить. Но я думаю, что нет смысла дрочить на абстрактную хуйню, не понимая общего контекста. Сейчас прочитаю простенький учебник по анализу на многообразиях, в котором в конце есть когомологии. Как раз узнаю что это и зачем они нужны.
#86 #377514
>>377512
Исправляют ошибки, добавляют упражнения, добавляют материал (могут целые главы добавить).
#87 #377515
>>377512
>>377514
Алсо, ты что, ебанутый? Не проще скачать и посмотреть в чем разница чем на дваче спрашивать?
>>377516
#88 #377516
>>377515
>>377514
Я верю двачу больше чем себе.
Плюс 13 издание я в сети не нашел, а 12 валяется на первой странице гугла.
>>377517
#89 #377517
>>377516
gen.lib.rus.ec
>>377519
#90 #377519
>>377517
ЧТо то там нету того что надо.
#91 #377527
Только что бросил читать на половине Постникова про теорему Ферма и алгебраические числа. Описанная там теория Кумера оказало влияние на появление таких терминов как идеал и дивизор, ну и связанных с ними теорий. Но меня это все равно не мотивировало на полное прочтение. Чувствую себя говном. Так ли я плох или может сама теория --- устаревшая архаика (~200 лет)? Как мне пережить то, что я не смог прочесть эту книгу до конца? Это ведь не я плохой? У вас ведь тоже так было?
#92 #377531
>>377514
Да хуйня это, отговорки. Единственная цель этих переизданий - бабла срубить авторам. Борьба с торрентами, только и всего. Но борьбу то все равно проигрывают поэтому уже давно прибегают к другой тактике - дофига книжек продаются с пасскодом для "домашки онлайн" или еще какой-нибудь такой хуйни. Получается похуй если ты купил б/у учебник задешево или скачал его, будь добор купить пасскод.
#93 #377534
>>377527
У меня так было с занятиями йогой. Начал было но застеснялся и забросил. Коврик себе купил, все дела.
#94 #377535
>>377527
Читал Маклейна, каротьш. Там такой пиздец лютый, ну я чет воспламенился и дропнул. Было досадно Потом опять, кстати, начал, но снова онхолднул, правда без дропа - прост нашёл чуть-чуть более удобоваримые книжки для начала, а потом может и адвансед стафф из Маклейна лучше пойдёт Ещё Зорича читал и даже не дропнул, а просто в какой-то момент забил, поняв, что гамалогии - сила, картофан - магила.
>>377538
#95 #377538
>>377535

>каротьш


Что, простите?
>>377539>>377574
#96 #377539
>>377538
Ну чево ты начинаешь, нормально же общались)
#97 #377542
>>377527
Потом может вернешься. А так ТЧ ИМХо скучнейшая область математики.
#98 #377545
Вопрос больше по теории вероятности, но с математикой связан. Итак, мы знаем, что если некоторое событие может произойти с равной вероятностью в любой точке некоторой ограниченной области пространства, то вероятность того, что оно произойдет в некоторой части этой допустимой области равна отношению площади этой части к площади области всех возможных точек.

И тут возник спор вероятность попадания в одну из точек. Чем он равен? Мне утверждают, что ноль, потому что площадь точки равна нулю. Но разве это верно?
>>377546>>377560
#99 #377546
>>377545
Да, объясню суть вопроса ещё немного.

То есть, для примера - попадание в половину отрезка равно 1/2, потому что отношение той площади отрезка, куда нам надо попасть и всех возможных вариантов. А вот что если надо попасть в точку. В одну точку из бесконечного числа точек некоторой ограниченной области. Площадь одной точки, как мне утверждают - ноль, потому и вероятность ноль. Но по логике теории вероятности - это недостоверное событие.
>>377559
#100 #377559
>>377546
Ты путаешь необходимость с достаточностью. Вероятность невозможного события равна нулю. Но из того, что вероятность события равна нулю, вовсе не вытекает, что оно невозможное. Идея, что событие нулевой вероятности - невозможное, ошибочна. В эксперименте могут произойти события, вероятность которых нулевая.
441 Кб, 1017x851
#101 #377560
>>377545
Да, вероятность попадания в данную конкретную точку равна нулю. Пикрелейтед (с.в. - случайная величина, д. - дискретная, н. - непрерывная).
>>377561>>377563
#102 #377561
>>377560
То, что надо было. Спасибо тебе.
#103 #377563
>>377560
И да, что за учебник, если не секрет?
>>377565
#104 #377565
>>377563
Письменный, конспект лекций по теории вероятностей, статистике и случайным процессам.
>>377566>>377568
#105 #377566
>>377565
Огромное спасибо.
#106 #377568
>>377565
Раз уж тут о тервере разговор, не ответишь на >>377487 ?
>>377569
#107 #377569
>>377568
http://new.math.msu.su/content_root/programs/kaf/matstat.htm
Программы мехмата. По ссылкам - документы со списком учебников в конце.
>>377570
#108 #377570
>>377569
Благодарю.
#109 #377574
>>377538
Каротыш. Коротышка тип.
#110 #377582
Меня поражает как люди что-то замечают. Читал начала теории групп в Винберге. Там ввели понятие смежных классов, потом оттуда так естественно вытекает теорема Лагранжа (вообще доказывать нечего), из нее куча очевидных следствий, в том числе из нее следует теорема Эйлера (нужно только рассмотреть правильную группу). Вчера прочитал, сегодня утром встал, на листочке перевывел - так это легко! А ведь наверняка поначалу люди долго ебались, чтобы это придумать.

Вот меня удивляет, что можно какой-то кусок математики систематизировать так, что довольно нетривиальные результаты можно получить на 2х страницах и это будет выглядеть очень естественно и легко. Причем я не просто поверил каким-то умным людям, а посмотрел, и мне все видно. Магия какая-то. Жаль, что в матане такого не бывает.
>>377590>>377620
#111 #377590
>>377582
Толсто.
>>377602
#112 #377592
Ребята, такой вопрос.
Читал вот учебничек по линалу, а там задание "покажите, что матрица В является обратной для А". Понятное дело, надо искать произведение BxA=E. Вот я его нашел. Могло ли в теории получиться, что AxB≠Е?
Просто не хочется лишний раз надрывать жопу и считать второе произведение.
>>377597>>377599
#113 #377597
>>377592
Для неквадратных матриц могло быть.
Для квадратных нет.
>>377606
#114 #377599
>>377606
#115 #377602
>>377590
Толсто в смысле про матан, или ты не веришь, что кто-то таким простым вещам может радоваться?
#116 #377606
>>377597

>>377599
Спасибо.
#117 #377620
>>377582
Ну-ка, если все так просто сделай видос и покажи мне что ты там доказываешь. Читать Винберга мне западло. А ты уже все понял. Можешь и ирл рассказать, готов приехать послушать если ты из дс/околодс.
>>377621
#118 #377621
>>377620

> Читать Винберга мне западло


С таким подходом мне что-то не хочется тебе ничего рассказывать.
>>377622
#119 #377622
>>377621

>мне что-то не хочется тебе ничего рассказывать


Такова суть любого матано-петуха. Что и требовалось доказать.
>>377623
#120 #377623
>>377622
Но я не люблю матан, я люблю алгебру.
>>377624
#121 #377624
>>377623
В том то и суть, что ты нихуя сказать не можешь по существу. Тебя попросили сказать чтоты узнал, а ты в духе "ДА Я ТЕБЯ СВОИМ КАРОННЫМ ПАХАРОННЫМ, ПРОСТА СВЯЗЕВАТСЯ НИАХОТА" слился.
>>377626
#122 #377626
>>377624
Так а зачем мне что-то кому-то доказывать на анонимном форуме? Все что хотел, я уже сказал.
#123 #377628
>>377293 (OP)
Уже второй трейдж я вас посещаю, и всенепременно вижу эту шапку. Меж тем эта шапка рикому в треде не нравится, о чем всегда мимокрокодил не приминет сказать. Либо вы тут балбесы, либо болтуны
>>377640
#124 #377634
Посоветуйте, кто сможет, годный учебник по основам комбинаторики и теории чисел для 1-2 курса. Буду очень благодарен любой инфе по этой теме
>>377639
#125 #377639
>>377634
Конкретная математика кнута мб
#126 #377640
>>377628

>шапка не нравится


Почему тебе не нравится шапа?

>Либо вы тут балбесы, либо болтуны


И то, и другое.
#127 #377643
Поясните мне за комплексные числа. Я понял что это расширение поля вещественных чисел, но что с ними делать?
Вон в википедии написанно что комплексные числа это числа решения уравнений, график которых не пересекает ось иксов. Как это понимать? Что почитать чтобы понять?
#128 #377646
>>377643

>комплексные


На курсере есть ща оч хороший курс по кмоплексному анализу. Тебе хватит лекций первой недели, чтобы ответить на все свои вопросы. Но надо будет въябывать.
>>377647>>377652
#129 #377647
>>377646
Ссылку в студию.
>>377651
#131 #377652
>>377646
>>377643

>расширение поля


>что с ними делать


>курс по кмоплексному анализу



Как же вы себе жизнь усложняете... В практическом применении комплексные числа очень просты для понимания. Уж точно можно понять "что с ними делать" без заморачивания полями и компланами.
>>377654>>377655
#132 #377654
>>377652
Физик-кун? Расскажи, где они там в физике, так чтобы прямо не обойтись. Интересно чисто, слыхал что такое есть.
>>377658>>377659
#133 #377655
>>377651
Ладно, буду курить.

>>377652
Ну, я из википедии все взял.
Я так понял, поле это все числа которые можно "взять в руку" и которые можно собрать в кучу введя пару простых правил.
Ну и поясни мне тогда что там такое с ними особенного, раз там все так просто.
>>377658
#134 #377658
>>377654
>>377655
Проще всего думать про них как про вектор. Просто вектор, в прямоугольных или полярных координатах. По одной оси Re по другой Im. Такая модель используется например в электричестве (переменный ток, импеданс вот этот все). В синусах косинусах считать неудобно, поэтому используются комплексные числа. В полярной форме их легче умножать, в прямоугольной - складывать. Потом переходишь обратно к синусам и получаешь результат. Вот и все.
>>377659
#135 #377659
>>377658
Ну вот. Я >>377654 не про это спрашивал, а про случаи, где в физике не обойтись без комплексных.
>>377660>>377666
#136 #377660
>>377659
Ну, физика. Квантувую механику кури тогда. Там не обойтись, мало не покажется. Я думал вопрос был просто "что с ними делать" на простом примере типа импеданса и фазоров.
#137 #377666
>>377659
Кстати не обойтись в КМ без комплексных чисел вовсе не потому, что это какая-то физическая сучность. Просто Шредингер так придумал. Он долго хуй к носу подгонял и колебался. В итоге пришел к уравению первого порядка по времени. И пошло и поехало, так что конечно уже не обойтись. Но приглядись - если из гамильтониана убрать V (потенциал не зависит от времени), то оно сведется к двум уравнениям - от координаты и от времени, причем последнее будет иметь вид dФ/dt = aФ. Понятно что тогда для осциллирующего решения а должно быть комплексным числом. Для сравнения, в классической физике уравнение колебаний струны - второго порядка (как по времени так и по координате) и его решением является действительная функция. Короче говоря, точно так же как и в электричестве, переход к комплексным числам в КВ - это трюк для удобства вычислений. Сам Шредингер признал, что если бы комплексные числа были необходимы В ПРИНЦИПЕ, это означало бы что состояние системы описывается какими-то двумя волновыми функциями (действительной и мнимой) которые нужно принципиально рассматривать вместе, что есть полная тащемта хуйня.
>>377667>>377708
#138 #377667
>>377666

>В итоге пришел к уравению первого порядка по времени.


Искалось, с самого начала уравнение первого порядка по оператору дифференцирования по времени( просто из-за принципа суперпозиций)

>Сам Шредингер признал


Ссылку

>что состояние системы описывается какими-то двумя волновыми функциями (действительной и мнимой)


Какой-то бред городишь, сама волновая функция физического смысла не имеет, вот квадрат ее модуля, это да.
>>377704
#139 #377668
Может ли тут кто проконсультировать меня по учебнику физики для абитуры? У меня серьезная проблема с одним учебником, я в панике
>>377680
650 Кб, 1460x1539
#140 #377669
Объясните дауничу, почему в примере 4.4 в нули сворачиваются две нижние строки, а в примере 4.5 там не в нули.
>>377670>>377671
#141 #377670
>>377669
Прибавь к третьей строке вторую, умноженную на -1.
Прибавь к четвертой строке вторую, умноженную на -2.
>>377674
#142 #377671
>>377669
Элементарно, в 4.4 2 строчка такая же как и 3 и 4, таким образом из 4 вычитают 3, а потом из 3 вычитают 2.
>>377674
#143 #377674
>>377670

>Прибавь к четвертой строке вторую, умноженную на -2.


Вот тут не въехал, там же другое получается, не?

>>377671

>таким образом из 4 вычитают 3, а потом из 3 вычитают 2.


Но разве тогда четвертая строка не примет вид второй?
>>377675>>377677
#144 #377675
>>377674

>Вот тут не въехал, там же другое получается, не?


Допер, ты про 4.4, а я думал про 4.5
#145 #377677
>>377674
На последнем шаге из 4 строчки вычли 3 умноженную на два И из 3 строчки вычли 2
#146 #377679
Спасибо за помощь, короче мне надо больше практиковаться с этими вычитаниями.
#147 #377680
>>377668
...
Может есть где то физики тред?
Нашел хорошую книгу косаткиной Физика для старшеклассников и абитуриентов: интенсивный курс подготовки к ЕГЭ, с темами объяснения законов на понятном языке и все нужные формулы
После каждой темы идут задачи по этой теме со сложностью A, B и C.
Но эта книга 2012 года и егэ там все 2012 и раньше. Не старо ли? Не проебусь ли я?
#148 #377681
>>377680
В физике отменили часть А.
>>377682>>377685
#149 #377682
>>377681
лул с какого года?
мимикрокодил
>>377692>>377693
#150 #377683
>>377680
Теорию из этой книги можешь взять, но задачник свежий нужен.
>>377685
#151 #377684
>>377680
Скинь скриншот страницы с вариантом, скажу тебе пойдет или нет.
абитура2016
>>377685
96 Кб, 626x524
85 Кб, 651x528
83 Кб, 643x462
105 Кб, 656x530
#152 #377685
>>377681
Да не в этом дело, задания то остались.
>>377683
Смотрел задачники, они даже не по темам, тупо кимы, а мне бы хотелось поступательно все решать, а не сначала всю теорию, а потом только решать, я так многое забуду да и скучно будет.
>>377684
Вот, это КИНЕМАТИКА, дальше динамика и статика и такая же проверочная работа.
И так ~700 страниц все темы. Более свежих аналогов даже других авторов не нашел..
88 Кб, 634x564
#153 #377686
>>377687
#154 #377687
>>377686
>>377685
Ну что бы было понятно: там сначала идет вся теория подразделов, а потом прорешевание всей части на разных уровнях сложности.
#155 #377691
>>377685

>>377685
Часть из этих могут попасться в первых заданиях, но ощущение, что много левой хуйни.
>>377695
#156 #377692
>>377682
С этого.

>>377685
На самом деле не остались.
>>377695
#158 #377695
>>377692
Бля и что делать? Ахуеть конечно, я думал это вброс, хотя бы примерные варианты выложили.
И что теперь, только B и С будет? Тоесть только средней и высокой сложности? Только их решать? Даже на решуегэ изменений нет.
>>377691

> могут попасться в первых заданиях


Да так как это первые темы.

>что много левой хуйни.


видимо так и есть(
>>377701
#159 #377699
>>377693
проиграл в гiлос
>>377700
#160 #377700
>>377699
Что не так?
#161 #377701
>>377695
В твоем задачнике конкретно один вариант сколько заданий содержит? В реальном ЕГЭ 32 задания.
>>377702
#162 #377702
>>377701
Да нет там вариантов. Там берется из сборников задач только определенные задания после пройденной темы.
Да уже все равно, вон какой пиздец через год на физике будет, буду решать тогда только B и С задания. А что еще делать? Вот именно что хуй знает.
>>377706
153 Кб, 592x305
#163 #377704
>>377667
Ты доебываешься до формулировок, я же не доклад пишу. Если хочешь from the horse's mouth, это есть в книжке, перевод шредингера с немецкого, называется Collected Papers on Wave Mechanics: Third Edition By Erwin Schrödinger стр 123. Поразительно, он говорит слово в слово тоже самое что и я, так что доебывайся до него.
>>377705
#164 #377705
>>377704
Окей ссылочку посмотрю, благодарю
#165 #377706
>>377702

>Да нет там вариантов


Хуйня какая-то. Бери нормальные варианты и смотри, а то на ЕГЭ шары вылупишь.
>>377707
#166 #377707
>>377706
Ты не понял не меня, не задачника. задания из ВАРИНАТОВ, только все в порядке по темам, а не в разброс как в обычных в вариантах. Кароче забей.

>Бери нормальные варианты и смотри


Даже книжки 2017 года для подготовки по физике говно, в них везде тестовая часть. А как блять ебанный егэ по физике будет выглядеть без тестовой части? вот хуй знает как именно.
#167 #377708
>>377666
А сейчас рассматривают маняфункции, которые имеют смысол только при возведении в квадрат.
Десять гауссов из десяти на кончиках декартов.
#168 #377709
Лучший учебник по теории множеств по мнению Двача? На английском.
>>377712
#169 #377712
>>377709
Jech, Set theory. 3rd millenium edition.
>>377713
#170 #377713
>>377712
Почему именно он?

Я сам теорию множеств то знаю, мне не для себя.
>>377715>>377717
#171 #377715
>>377713
Потому что СКОЗАЛ
#172 #377717
>>377713
Он по-моему сбалансирован. Покрывает много материала из современной ТМ, написан по делу, без лукавых мудрствований, некоторые доказательства оттуда более лаконичны и прозрачны, чем в более старых работах. ХЗ.
#173 #377722
Кстати, а есть ещё в интернете какие-нибудь ебанутые мат. программы? Типа Вербицкого.
>>377725>>377732
#174 #377725
>>377722
Оп-пост этого треда
>>377727
#175 #377727
>>377725
Так это даже не программа, так, свалка книг.
#176 #377728
Почему в шапке книг от ОПа нет Бурбаки!?
>>377735
#177 #377732
>>377722
Программа мехмата.
#178 #377735
>>377728

>Бурбаки


не нужен
на капче 666
>>377736
#179 #377736
>>377735
666 из хуев не так зловеще
#180 #377738
>>377643
Проще всего представить комплексные числа как поле линейных многочленов по модулю x^2+1.
>>377747>>377754
#181 #377747
>>377738
Чёто плоскость попроще.
>>377748
#182 #377748
>>377747
Плоскость не объясняет как перемножать комплексные числа
>>377754
#183 #377754
>>377738
Толсто. Нарисовал вектор-палку и "представил". Как еще можно проще представить.

>>377748
Полярные координаты объясняют. И формула эйлера. Из полярных координат легко переходить к декартовым и наоборот. Куда еще проще.
>>377757>>377920
#184 #377757
>>377754

>Полярные координаты объясняют. И формула эйлера. Из полярных координат легко переходить к декартовым и наоборот. Куда еще проще.


Ничуть не проще. Так же. Алгебраический подход очень удобен тем, кто ок с алгеброй.
#185 #377870
Меня просят нарисовать график функции x^2+y^2=<3
Это что, мне надо зарисовать весь кружок в радиусе 3 от центра координат!?
>>377873
#186 #377873
>>377870
Нет. Радуса корень из трёх.
>>377876
#187 #377876
>>377873
Ну не важно. Зарисовать весь кружок!?
>>377878
#188 #377878
#189 #377879
В упражнениях по математике у меня получается результат уровня корень из 20. Это так и задуманно или я где то ошибся?
Задание узнать расстояние между двумя точками по теореме пифагора про штаны.
>>377880
#190 #377880
>>377879
Нормально.
#191 #377920
>>377754
Если всё так просто то найди при помощи векторов или полярных координат произведение двух комплексных чисел (a, b) и (c, d).
При помощи многочленов всё получается просто:
(a + bx)(c + dx) mod (x^2+1) = bdx^2 + (ad +bc)x + ac mod (x^2+1) = (ad + bc)x + ac - bd
Это может понять любой семиклассник
#192 #377922
>>377920

>полярных координат


r1 * r2 exp(i\phi1 + i\phi2)
>>377924>>377926
#193 #377924
>>377922
Из чего следует твоя формула? Где вывод, как в >>377920?
>>377925
#194 #377925
>>377924

>Из чего следует твоя формула?


Смотри в любом учебнике по комплексному анализу(даже по математическому, в тех курсах, где комплексные числа затрагиваются)

>Где вывод, как в >>377920?


Я бы написал, но мне пиздец как лень рисовать очевидные вещи, оставляю тебе в качестве упражнения.
>>377930
#195 #377926
>>377922
И ты путаешь показательную форму и полярные коородинаты
>>377927
#196 #377927
>>377926
Я хз что тот анон имел в виду под полярными координатами, в моем понимании это то, что я написал.
>>377945
#197 #377930
>>377925
Проблема в том, что ты пытаешься вывести своё умножение из умножения экспонент с комплексным показателем, то есть объяснить непонятное неизвестным.

Бро с R[X]/(X^2+1) двощую и одобряю.
>>377931>>377945
#198 #377931
>>377930
Я мог бы с тем же успехом нарисовать косинус и синус вместо экспоненты. Как известно под этой записью имеется вполне очевидная геометрическая подоплека.
9 Кб, 917x55
#199 #377933
Как это сделать правильно?
>>377934
#200 #377934
>>377933
Уножить на 4. И вычесть.
#201 #377935
Читаю калькулус томаса.
Дошел до 50 страницы. Полет отличный. Проблем только в терминах, уровня точка начала координат у пиндосов называется оригинс и подобные.
>>377936>>377970
#202 #377936
>>377935
вкинь ссылку на книгу ради лулза
#203 #377937
>>377939>>378075
#204 #377939
>>377937
мне вот интересно швятых западнецов вообще в школе блять чему учат
>>377940>>377946
#205 #377940
>>377939
Что не так?
>>377941
#206 #377941
>>377940
ну когда в учебнике поясняют что такое знак суммы сигма, это такое. Интеграл Римана тоже неправильно введен судя по всему.
>>377942
#207 #377942
>>377941
Скрин давай где это там такое вводится и что там неправильно.
>>377943
#208 #377943
>>377942
страница 263 из которой следует что интеграл римана вводится как предел римановских сумм при n->\inf но всем известно что ТАКЖЕ шаг разбиения должен при этом идти в нуль, у томаса я этого не вижу.
>>377944
#209 #377944
>>377943
отбой, посмотрел выше, там поясняется
>>377950
#210 #377945
>>377927
Что значит хз. rcosЭ + isinЭ.

>>377930
Да, через косинус синусы/ триг identities вывод кучерявее получается, и что. Зачем подменять понятия. Степень сложности выводимости не имеет никакого отношения к простоте восприятия. Вопрос же был в том, как проще ПРЕДСТАВЛЯТЬ а не выводить правила действий над ними.
>>377947
52 Кб, 620x490
#211 #377946
>>377939
начинается блядь.
>>377948>>378030
#212 #377947
>>377945

>Что значит хз


Это называется тригометрическая форма.

>как проще ПРЕДСТАВЛЯТЬ


Тут я на стороне формалистов, в мире много вещей которые не имею "представления" в житейском смысле этого слова.
>>377953
#213 #377948
>>377946
Просто непривычно видеть, чтобы в учебнике пояснялся значок суммы или что то подобное, это довольно странно для меня.
#214 #377950
>>377944
Вот вот, не надо тут. Учебник топовый.
>>377952
#215 #377951
>>377948
Ну ты алёша.
#216 #377952
>>377950
Попадись он мне на первом курсе, матан прошел бы легче наверное, хотя в конце концов и фихтенгольца хватило заглаза
>>377956
#217 #377953
>>377947
Но сам же апологет алгебраического подхода сформулировал свою мысль так:

>Проще всего представить комплексные числа как поле линейных многочленов по модулю x^2+1.



То есть мы о представлении говорим или о чем?

>>377948
ну если ты в математику можешь и в логику наверное, зачем обобщаешь как баба срака
>>377955>>377963
#218 #377955
>>377953

>ну если ты в математику можешь и в логику наверное, зачем обобщаешь как баба срака


Изначально был негативно настроен.

>То есть мы о представлении говорим или о чем?


В общем, кому-то проще осознавать через линейные многочлены по модулю, кому-то через показательную форму, вопрос вкуса.
>>377957>>377963
#219 #377956
>>377952
У него просто все поясняют подробно. Учебник написан так сказать с позиции дауна который хочет понимать лекции ромы михайлова и выебываться этим на сосачах.
>>377965
#220 #377957
>>377955
Просто меня слегка припекает, что многие почему-то не понимают, или не хотят понять, что калькулюс - это учебник для инженеров. Это не матан. Строгостей там не надо искать. Матану там учат по другим учебникам. См. шапку Ж)
>>377959
#221 #377959
>>377957
А ведь у них еще вроде есть и прекалькулюс. Да, все время почему-то идет отождествеление калькуюса с матаном у нас.
>>377960
#222 #377960
>>377959
Прекалькулус - это high school обычно, ну или рефрешер для ебланов в community college кто SAT неасилил чтобы им можно было calculus брать.
>>377961
#223 #377961
>>377960
Им только за щеку брать можно.
#224 #377963
>>377955
>>377953
Константная часть многочлена = Re часть, коэфф возле x = Im часть. В R[X]/(X^2+1) дополнительно видно, почему умножение именно такое, какое оно есть. Сразу виден и способ получения этого поля.

Что касается экспонент.. дык их и здесь можно выводить спокойно, также через ряды.
#225 #377965
>>377956
Помню, Рома в видосе рассказывал как у него в детстве не было интернета и он трехтомник Фихтенгольца выменял у друга.
#226 #377970
>>377935

> Полет отличный.


Еще бы. Этот учебник даже для школы слабоват. Такое классе в 6-7 еще норм читать, но где-то 8 классе уже пора завязывать с математикой без доказательств.
#227 #377989
>>377970
Доказательства не нужны.
t. Ландау
>>377992>>378043
#228 #377992
>>377989
Не мешай второкуру выёбываться перед первокурами))
>>377997
#229 #377997
>>377992
Не мешай третьекуру выёбываться перед второкурами))
#230 #378003
>>377948
Ненавижу лютой ненавистью таких, как ты. И не читаю русских учебников и русских статей по той же причине. Русские должны сдохнуть.
>>378004>>378005
#231 #378004
>>378003
Какой ты умный
#232 #378005
>>378003
При этом ты пишешь на русском, любопытный диссонанс, вас шизофрения случаем не мучает?
#233 #378025
>>377970

>математикой без доказательств


математика без доказательств не нужна
#234 #378026
>>377970
Щас нам этот анон пояснит зе лекции ромы михайлова.
#235 #378029
Где можно научиться более или менее практической теории групп? Ну например, для физиков, биологов, да хоть data scientist или ассенизаторов - не важно, главное посмотреть, какие у этой вещи практические применения. Или это исключительно в рамках искусства?
#236 #378030
>>377946
Что начинается? Западные общеобразовательные программы объективно слабее российских, патриотизм здесь не при чём.
#237 #378031
>>378030
Почему объективно? И какие ещё "западные общеобразовательные программы", ведь на западе такого явления вообще нет?
14 Кб, 320x240
#238 #378034
Что посоветуете годного по матлогике? Гильберт, Аккерман "основы теоретической логики", Карри "основания математической логики" - годнота, или есть что-то актуальнее?
>>378036
#239 #378036
>>378034
Это (а также книги Клини) - must read по дефолту.
#240 #378042
>>378030
Давай, обосновывай свои слова или тостейший лев когда либо заходивший на сци
23 Кб, 800x450
#241 #378043
>>377989
Всякому технобыдлу, безусловно. Нормальным людям нужны.

>>378030
Ну ТУПЫЫЫЕ
>>378045
#242 #378045
>>378043
Нормальный человек, докажи мне что косинус четная функция.
>>378046>>378049
#243 #378046
>>378045
Подставь туда x и -x. В чем проблема?
>>378047
#244 #378047
>>378046
И это доказательство!?
И этот пацак потом что то кукарекает про херовые доказательства в нижке!?
>>378055
#245 #378049
>>378045
Построй график косинуса и посмотри.
>>378052>>378054
#246 #378052
>>378049
Доказательство уровня маттреда
#247 #378054
>>378049
Вонючий школотун, как же хочется выпнуть тебя из треда пинком под зад.
#248 #378055
>>378047
Это был не я. А в чем проблема, собственно? Я определяю косинус через ряд, если че.
>>378057
#249 #378057
>>378055
Алсо, по первой же ссылке в гугле нашел
https://proofwiki.org/wiki/Cosine_Function_is_Even
Первое доказательство именно такое же как мое. В чем проблема-то?
#250 #378058
Когда читаешь книги Бурбаки, тебе открывается истина.
>>378060
#251 #378059
Вы глупцы, если не используете Бурбаки для обучения.
>>378060
#252 #378060
>>378058
>>378059
Он обманывает, не ведитесь.
>>378062
#253 #378061
>>378029
Неужто никто не знает?
>>378066
#254 #378062
>>378060
Я вполне серьёзно.
#255 #378066
>>378061
Нигде кроме маняматики не применяется.
Как ты себе вообще представляешь её применение? Группа электронов? Группа тел с массой эмжэквадрат?
#256 #378075
>>377937
Какой здоровый калькулюс
#257 #378079
>>378029
Применяется в современной физике.
306 Кб, 1123x2369
#258 #378081
Повторяю курс математики за среднюю школу. Запнулся на пикрил. Что я делаю не так? Не понимаю как разложить "1)"
И вопрос:
При построении x^2(2x^2 + 1) степени складываются, а при возведении многочлена в степень - перемежаются, так?
>>378083>>378084
#259 #378083
>>378081
У тебя во втором не так, в скобках там основание должно быть не 1, а 4. А в первом что непонятно? (x^2 - 1) можно ещё разложить на (x + 1)(x - 1)
>>378086
#260 #378084
>>378081
А, и во втором ещё в скобках тоже неправильно, должно быть не "- 4", а "- 2".
>>378086
#261 #378086
>>378084
>>378083
Мой почерк, увы, таков, что y за 4 можно принять. Там нет 4, есть y.
>>378088
#262 #378087
>>378029
погуглил group theory biology|chemistry|physics
не знаю если тебе это нужно

Review and application of group theory to
molecular systems biology
http://www.ncbi.nlm.nih.gov/pmc/articles/PMC3149578/pdf/1742-4682-8-21.pdf

MOLECULAR SYMMETRY, GROUP THEORY, & APPLICATIONS
http://vallance.chem.ox.ac.uk/pdfs/SymmetryLectureNotes2009.pdf
в самом верху есть список книг.
>>378182
#263 #378088
>>378086
Ну если заменить 4 на y в моих ответах, они останутся корректными.
>>378090
#264 #378090
>>378088
Спасибо
#265 #378095
А я напоминаю, что в качестве первой книги по математике вообще лучше всего подходит

> S. Axler: "Linear algebra done right".


Книга очень простая (первые несколько глав), но не для дебилов.
>>378096
#266 #378096
>>378095
Надо, конечно, добавить, что книга немного упоротая:
1) Обозначения немного нестандартные: null space вместо kernel, list вместо n-tuple и другие.
2) В некоторых ситуациях, когда можно рассматривать произвольное поле нулевой характеристики или вообще произвольное поле, там используется R или C.
3) Из-за того, что определители выпихнуты в последнюю главу, характеристический многочлен определяется через треугольную форму матрицы отдельно для матриц над C и отдельно для матриц над R.
Но в целом годнота, рекомендую.
#267 #378097
>>378030
Ты 15 рублевый что ли? "Западные программы". "Эти земляне такие тупые".
#268 #378099
>>378030
Базовые школьные по математике - да, слабее, но заинтересованный американский школьник с легкостью идёт на дополнительные занятия, превосходя ту школьную программу, что навязывается в российских мат. классах.
А вот высшего математического образования в России нет: оно застряло на уровне 1920-х (брать интегралы из Демидовича), а США имеет сотни ведущих математических высших учреждений, лучших в мире, где обучают современной математике.
666 на капче.
>>378100>>378105
#269 #378100
>>378099
Заинтересованный российский студент с легкостью идет на спецкурсы, превосходя ту университетскую программу, которая навязывается в американских бакалавриатах.
#270 #378101
>>378029
Элементарщина с конечными группами в простых алгоритмах на теорию чисел (тип обратное по модулю) + в группах галуа (например док-во неразрешимости уравнения в радикалах).

Они же уже существенно используются в комбинаторных играх (кубик рубика и почти любые другие), шифрах, задачах.

В физике по теореме нётер каждый закон сохранения = симметрия => группа симметрий. Группы симметрий это совсем другие группы, они большие и непрерывные, как группа лоренца или пуанкаре, теория совсем другая.

В алгебраической топологии основные инварианты -- абелевы группы. Гомотопические, гомологии, иногда когомологии.
>>378182
#271 #378102
>>378100
Нет.
>>378103
#272 #378103
>>378102
Пидора ответ.
#273 #378105
>>378099
Двачую. В отличие от традиций совка, там никто ничего не навязывает. Можешь просидеть high school c 9 по 12 класс и заниматься всякой хуйней, лепить дилдаки на уроках лепки из глины. Это и есть core aka "базовая хуйня если тебе все пох лишь бы выпустится и пойти на ферму работать". А можешь брать AP классы по математике, в тч за колледжные кредиты. То же самое дальше в колледжах, выбора еще больше, от матана для сварщиков до межпланетных мочедзук. Поэтому ссу на бинарных ебланов обобщающих про "западное образование". Тупые обмудки еще имеют наглость сидеть в маттреде, хотя даже в элементарную логику не умеют со своими однобитными мозжечками.
#274 #378106
>>378100

>программа навязывается


>в американских колледжах


Не пойму, ты тупой или тролишь так уныло?
#275 #378107
Наукач, пришли ко мне книги: Зорич, Хаггарти. Годнота, одним словом.

Теперь вот имею наглость попросить вашего совета.
По алгебре - Винберг или Кострикин. Или вообще что-то другое?
По геометрии думаю брать
Калинина.

Что по линалу что посоветуете?

Заранее спасибо.
>>378208
#276 #378112
>>378029
В геометрии изучаются свойства фигур относительно той или иной группы преобразований пространства.
>>378182>>378197
246 Кб, 1024x600
118 Кб, 1024x600
85 Кб, 1024x600
#277 #378113
Почему на первой пикче, задания 31 32, у меня получается разброс в значениях пропорциональности?
Почему две картинки суммы функций не совпадают? Или это просто пример как функции плюсуются?
>>378116
#278 #378116
>>378113
Почему они должны совпадать? Две разные картинки, разные функции, на первой +, на второй + . для двух других фций.
>>378117
#279 #378117
>>378116
Ну хорошо. А первая пикча тогда почему с ошибками? Или это ошибки для точности?
>>378122
#280 #378121
Аноны, что можно почитать по вложенным радикалам?
https://ru.wikipedia.org/wiki/Вложенные_радикалы
>>378214
#281 #378122
>>378117
Наверное там ключевое слово reasonably support the proportionality assumption. И дальше от тебя требуется estimate. То есть примерно 6. Не знаю, у тебя же учебник, может там речь идет об экспериментальных результатах и нужна примерная оценка? Это же для инженеров :)
87 Кб, 1273x322
#282 #378148
Охуенно! Какая-же тонкая ирония, про бесполезность упражнений со звёздочкой.
#283 #378158
>>378100
Приноси эти спецкурсы. Ведь должны же они существовать в природе, раз ты такое утверждаешь.
#284 #378168
Хуево считаю в уме, как апнуть этот скилл?2016-1988, к примеру, долго считаю
>>378169>>378181
#285 #378169
>>378168
странно, ты 1988 инстинктивно не дополняешь до 2000? ты сразу не видишь 12 которые складываешь с 16?
>>378177
#286 #378177
>>378169
Да, про это я и говорю. Всегда в математике были проблемы со счетом в уме. В принципе не особо мешает, даже в универе, но напрягает меня самого, лично.
#287 #378178
Кто-нибудь читал Высшую геометрию Ефимова? Какие у нее требования к предварительным знаниям (школьные стереометрию и планиметрию помню так себе)?
#288 #378181
>>378168
Такая же хуйня. В шкалке всегда считал на черновике. Даже двухзначные числа складывал. Боялся проебаться где-нибудь.
>>378257
#289 #378182
>>378087
>>378101
>>378112
Спасибо, парни.
#290 #378197
>>378112

>В геометрии изучаются свойства фигур относительно той или иной группы преобразований пространства.


Пиздеж теоретико-групповиков. Ни разу не видел, чтобы геометрию изучали с помощью групп.
>>378199>>378201
#291 #378199
>>378197
А вот теорию групп с помощью геометрии хорошо так изучают.
#293 #378204
>>378201
Пиз-дёж. Открой любой учебник по геометрии. Где там U(1)? Где там SO(2)? Старый добрый Евклид со своими аксиомами.
>>378220
#294 #378208
>>378107
Ты из какого века будешь? Книги читать в комплюктере.
>>378209>>378211
#295 #378209
>>378208
*можно
#296 #378211
>>378208
Бумажные можно перед сном читать, а за комплюктером сидеть надо. С другой стороны, можно планшет какой-нибудь купить или читалку, но мне как-то удобней лежа в кровати читать бумажную книгу.
>>378252
#297 #378214
>>378121
Ты аутист?
#298 #378220
>>378204

>Открой любой


Лелон-Ферран Основания геометрии, Берже Геометрии. Куда же пропал старый добрый Евклид?
>>378246
43 Кб, 555x340
101 Кб, 531x769
#299 #378225
я думаю здесь много компетентных людей, которые могли бы мне помочь. я не до конца понимаю обоснование строгости неравенства (1). пожалуйта, помогите.
>>378236
#300 #378236
>>378225
Нихуя не понял что тебе не понятно. Алсо что это за старинный учебник?
>>378239
#301 #378237
Помните, что форсом математики занимаются пидоры из издательств. Главная их цель продать вам за шекели свою ненужную макулатуру. Другой задачи у математики нет. Точно также, как читая книги Дугина по геополитике вы не станете политоком, так и читаю Зорича вы не станете математиком. Но деньги уже потрачены. Не ведитесь на форс.
>>378238
#302 #378238
>>378237
Читаю все книги бесплатно. Кто кого наёбывает в таком случае?
>>378251
#303 #378239
>>378236
это Г.М. Фихтенгольц "Курс дифференциального и интегрального исчисления". мне непонятно, почему из-за возможности увеличить а, а', b, b' пропадает знак равенства
>>378240
#304 #378240
>>378239
Это как спросить, почему параллельные прямые не пересекаются. Мы ВЫБИРАЕМ числа a, a', b, b' так, чтобы выполнялось неравенство 1. Числа, не удовлетворяющие неравенству, не рассматриваем.
236 Кб, 794x1029
#305 #378246
>>378220
Открыл. А где тут, собственно, геометрия? Я хочу задачи из ЕГЭ решать, зачем вы мне лемму Цорна принесли?
>>378253
#306 #378251
>>378238
Издательства тобой недовольны. Жди счет за нарушение копирайта.
#307 #378252
>>378211
Так вся суть чтения учебника - это быстрый поиск нужных мест. Это же не детективчик.
>>378285
#308 #378253
>>378246

>Я хочу задачи из ЕГЭ решать


Любую задачи из школьного курса можно решить методами линейной алгебры.
>>378271
#309 #378257
>>378181
Меня дедушка научил на 11 умножать двузначные числа - раздвигаааешь и вставляешь между ними их сумму. Например 23x11 = 253. Думал тянок этим фактом поражать, но им как-то пох.
>>378259>>378269
#310 #378259
>>378257

>на 11 умножать двузначные числа - раздвигаааешь и вставляешь между ними их сумму


Задачка для треда. В скольких процентах случаев методика деда дает верный результат?
>>378263
#311 #378263
>>378259
Чо-то около сорокета в процентах.
3 Кб, 340x135
#312 #378264
Методика деда дает 100% результат, но надо поменять условие с "раздвигаааешь и вставляешь" на алгебраически точное.
>>378267
#313 #378267
>>378264
Так это другое условие, ёбта. По методике деда, 48*11 == 4128.
>>378272
#314 #378269
>>378257

>Думал тянок этим фактом поражать, но им как-то пох.


Тянки текут от топологических векторных пространств, инфа 100%
>>378272
143 Кб, 2048x478
#315 #378271
>>378253
Никто тебя за язык не тянул.
Пикрелейтед.
>>378332>>378471
#316 #378272
>>378267
Это в военное время. А для small angles в смысле небольших значений методика деда отлично работает!

>>378269
Ну это слишком абстрактно. А тут умножаешь и получаешь, например, длину чего-нибудь важного. Например у тебя 11 инчей а ты хочешь перевести свой размер в сантиметры. Умножай смело на 25 и откидывай последнюю цифру. Огого!
15 Кб, 276x179
#317 #378278
У кого-нибудь есть книжки этого издательства?
#318 #378279
Часто встречаю подобную запись скорости выполнения О(1). На вики прочитал, более менее понял, но вот только практическое применение не осознал.

Вопрос в чем, О(1) и о(1) - это сколько секунд?
>>378280
#319 #378280
>>378279

> О(1)


Константа, независимо от размера входных данных. Если алгоритм программа обрабатывает вход за O(1), то она за одно и то же время обработает и килобайт, и петабайт.

>о(1)


Нуль или неразличимо малая величина.
>>378282
#320 #378282
>>378280
Спасибо за ответ.
#321 #378285
>>378252
Ну так для этого индекс и оглавление существует.
>>378297
#322 #378297
>>378285
Иной раз нужн вбивать конкретную фразу, причем в гугл даже а не в учебник, и многократно ее менять чтобы выскочило конкретно то что ищешь, например в google books. Это метода 2016. Мой дед так не умеет например. У него только бумажный фихт 1966 года издания.
>>378315
#323 #378315
>>378297
Срыв покровов: иногда нужно быстро нагуглить что-то по фразе, а иногда люди читают книги подряд.
>>378346
#324 #378332
>>378271
Посоны, помогите решить эту задачку алгебраическими методами. Вроде все просто, но чет не получается пока.
>>378334
#325 #378334
>>378332
Расскажи что ты делаешь и что конкретно не получается.
>>378342
#326 #378335
Ананасы, объясните, пожалуйста, гомотопии кривых. Везде непонятно. Какие-то прямоугольники, переводящиеся непрерывным отображением в кривые. Я у мамки не математик.
>>378418
35 Кб, 909x815
#327 #378342
>>378334
Пока не знаю как выразить координаты точки касания с наклонной линией. Радиус должен быть чуть меньше 3. А значит там в ответе будут или радикалы или арксинусы.
>>378436
#328 #378346
>>378315
Но это тупо, читать учебник от корки до корки как худ лит. Да даже если читаешь, все равно нужно бывает быстро откинуться назад и найти ключевые места.
>>378372
#329 #378372
>>378346

> Но это тупо, читать учебник от корки до корки как худ лит.


Во-первых, читать подряд не означает читать от корки до корки. Можно пропускать параграфы или даже целые главы, если они кажутся неинтересными.
Во-вторых, не нравится читать подряд - не читай. Че доебался-то?
>>378444
10 Кб, 682x74
#330 #378416
Читаю Алуффи, встречаю вот такое определение отношение эквивалентности над Z для получения группы Z/nZ, нихуя не понял, как это читать. Интуитивно, что в одном классе эквивалентности должны быть числа с одинаковым остатком от деления на n, но почему бы так и не записать, что остаток одинаковый? Почему даётся не эквивалентность двух чисел a и b, а эквивалентность a и остатка от деления?
>>378418
176 Кб, 220x161
#331 #378418
>>378416

>почему бы так и не записать, что остаток одинаковый?


Потому что это понятие нужно отдельно вводить. Проще и логичнее сказать, что a и b принадлежат к одному классу mod n если их разность кратна n.

> Почему даётся не эквивалентность двух чисел a и b, а эквивалентность a и остатка от деления?


Даётся именно эквивалентность двух чисел a и b. Просто запись такая. "а равно b по модулю n".

>>378335
Кривая в пространстве X = непрерывное отображение [0,1]->X.

Если одну кривую в другую можно "плавно" (=непрерывно по всем параметрам) перевести, то они гомотопичны.

Гомотопичность относительно концов кривых -- это когда все рассматриваемые кривые должны иметь фиксированные концы (пикрелейтед гомотопия с фикс концами).

Какую книгу читаешь?
>>378419>>378475
#332 #378419
>>378418

>Просто запись такая. "а равно b по модулю n".


Вот, теперь всё ясно, спасибо.
#333 #378436
>>378342
Появилась новая идея как решить эту задачку. Все точки начала окружности походу лежат на одной прямой. А вектор от точки касания до точки начала окружности ортогонален линии касания.
По сути у нас будут два вектора:
(0,0);(x_точка касания, y_точка_касания)
(x_начало_окружности,y_начало_окружности);(x_точка касания, y_точка_касания)
Берем скалярное произведение и оно равно нулю.
Как это выразить формульно пока не знаю.
Посоны, помогите.
#334 #378444
>>378372
Да не доебался, нормально общаемся же. Тред надо поднимать, он на второй строчке болтается второй день. А должен быть в тройке всегда.
#335 #378447
Ебать я лох, сортировка как-то переключилось на время создания вместо бампов. Все в порядке, в тройке.
#336 #378450
С компа матлитературу читать неудобно
>>378469
#337 #378469
>>378450
Мне удобно. Две вкладки одинаковые открываю, одну для собственно чтения и одну для референсов
>>378483
39 Кб, 1251x1038
#338 #378471
>>378271
Радиус внутри (2.95...3). Точное значение пока не могу найти. Помогите составить уравнение скалярного произведения.
21 Кб, 958x892
#339 #378474
Решил эту задачу. Ответ: (-8 + Sqrt[194])/2
>>378476
#340 #378475
>>378418

>Кривая в пространстве X = непрерывное отображение [0,1]->X.



>Если одну кривую в другую можно "плавно" (=непрерывно по всем параметрам) перевести, то они гомотопичны.



>Гомотопичность относительно концов кривых -- это когда все рассматриваемые кривые должны иметь фиксированные концы (пикрелейтед гомотопия с фикс концами).



>Какую книгу читаешь?



То есть это как бы двумерное отображение? Где-то видел, что гомотопию деформацией называют. Только зачем для этого вводится прямоугольник, одна из сторон которого отображается в одну кривую, противоположная - в другую?

Читаю, в основном, интернет. Наслышан, что книга Рунде неплохая, но взяться за чтение математики на бусурманском языке пока немного очкую.
>>378480
#341 #378476
>>378474
а второй ответ?
>>378489
10 Кб, 742x53
#342 #378478
Как переводится здесь слово fiber? Переводчик говорит про какие-то волокна.
#343 #378479
>>378478
И вот еще: как переводится a fiber is singleton?
>>378485>>378555
#344 #378480
>>378475

>Только зачем для этого вводится прямоугольник, одна из сторон которого отображается в одну кривую, противоположная - в другую?


Это просто способ рисовать, как устроено отображение гомотопии.

Ведь как ты эту деформацию задашь? Деформация будет функцией от времени от t = 0 до 1, на каждом значении t задающая свою кривую [0,1]->X. То есть некая g(t, x) : [0,1]x[0,1]->X, которая на t=0 одна кривая, на t=1 другая. Это и обозначают рисуя прямоугольник. Хуй знает, мне кажется и так понятно, что такое гомотопия, без дурацких прямоугольников. В английской википедии например нормальное описание, без всяких прямоугольников долбоебических)
>>378488
#345 #378481
>>378478
Вроде никак не переводится. У нас говорят "прообраз" или вроде того.
>>378482
#346 #378482
>>378481
В книге какого-то препода из НМУ подобная штука называлась слоем, но тут переводчик про какие-то волокна говорит.
#347 #378483
>>378469
Двачую, так же делаю. У меня еще 2 моника, один 27 дюймов, другой 20.
#348 #378485
>>378478
>>378479

> fiber of f over q


Прообраз q при отображении f.

Fiber - это просто fancy word. Обычно говорят inverse image of {q} under f, а по-русски: прообраз {q} при отображении f.

> fiber over q is a singleton


Прообраз q состоит из 1 элемента.

Вообще, забей на переводы. Русские термины очень быстро выяснятся по ходу дела, когда ты начинаешь читать что-то на русском. Никаких проблем от незнания русских терминов у тебя не будет.
#349 #378486
fiber это preimage же %)%
#350 #378487
А, ещё "fibered product" переводят как "расслоённое произведение"
#351 #378488
>>378480
Спасибо.
28 Кб, 1235x987
#352 #378489
>>378476
Никакого второго ответа я там не вижу. Вариант с окружностью внутри треугольника условием задачи не предусмотрен.
#353 #378503
Поцоны, чето хочу топологию поизучать, но применять никаким хуем не буду, то есть для себя. Сам не математик нихуя. Так вот, топология интересна или мне идти на хуй?
>>378508
#354 #378505
И ищо вопрос вдогонку. Чего вообще в матеше такого пиздец интересного, что люди занимаются ею всю жизнь? Понимаю еще матеша на стыке с физикой, там хотя бы какую-нибудь парашу описать из ирл жизни можно. Но полностью абстрактная матеша - это ж пиздец, никакого соединения с реальностью. Нахуй так жить?
#355 #378506
>>378505

>никакого соединения с реальностью


В этом соль, кагбэ. Реальность - хуита.
#356 #378507
>>378505

>матеша.


где ты такое слово выцепил дебильное пиздец. иди лучше днюху справляй с пацанами. правильно понял - лучше идти на хуй.
>>378510
#357 #378508
>>378503

> Так вот, топология интересна или мне идти на хуй?


на, изучай. Потом решишь - интересно, или нахуйвпизду.
http://gen.lib.rus.ec/book/index.php?md5=5CD3A3DB7FC638EAC684359D7551A036
>>378510
#358 #378510
>>378508
В каком-то факе видел эту книгу - Khetcher_Algebraicheskaya_Topologia_2011. Че пижже будет, твоя ссылка или эта книженция?
>>378507
Хуя агрессивный. Тебе не похуй?
52 Кб, 445x369
#359 #378511
>>378505

>Чего вообще в матеше такого пиздец интересного, что люди занимаются ею всю жизнь?


Теория чисел. Самый годный раздел математики
>>378513
#360 #378512
>>378510

>Khetcher_Algebraicheskaya_Topologia_2011


Проиграл. Эта книга для graduate students. Ты там даже 2 страниц не осилишь.
>>378514>>378537
#361 #378513
>>378511
Не слушай его, это наёб
>>378518
#362 #378514
>>378512
То, что ты дал - энтри левел? Тогда норм.
#363 #378516
>>378510

>Че пижже будет, твоя ссылка или эта книженция?


моя научно-популярная для даунов. Это чисто почитать для общего развития, не учебнтк.
>>378517
45 Кб, 1020x112
#364 #378517
>>378516
Это автор про додекаэдр чтолей?
#365 #378518
>>378513
Вот этого не слушай, самый охуенный раздел.
>>378519
#366 #378519
>>378518
Хуль там такого интересного? Вики ничего такого охуенного не рассказывает, так что давай ты.
>>378521
#367 #378521
>>378519
Я сам пока в неё вкатоваюсь, читаю Прахар Распределение простых чисел.
Просто меня удивляет, сколько всего скрывается за такими простыми вещами, как натуральные числа.
>>378522
#368 #378522
>>378521

>сколько всего


Конкретнее. Что там такого интересного?
>>378523
#369 #378523
>>378522
Например, почему одни числа содержат в разложение на простые множители только два множителя, а другие три. Почему блядь?! Как так происходит? И почему именно эти числа?
v(n) - количество простых множителей.
v(4)=v(6)=v(10)=v(15)=2, почему именно эти числа? Есть ли какай-нибудь общая формула для таких чисел? И сколько их на отрезке [1;n]?
Тоже и для чисел a, таких, что v(а)=m. Тебе не кажеться это интересным?
>>378524>>378526
#370 #378524
>>378523
Вот уж в хуй не впилось.
>>378525
#371 #378525
>>378524
А что тогда тебе интересно?
>>378527
#372 #378526
>>378523
Хуя тебя понесло. Меня такая хуита интересует в физике, в связи времени и пространства. Только там нет пруфов и одфа философия.
#373 #378527
>>378525
Теория типов, например
>>378528
#374 #378528
>>378527
Это что-то программеское? Или связанно с гомологиями?
>>378529
#375 #378529
>>378528
И да, и нет. Меня оно интересует как альтернативные основания математики тащемта
#376 #378532
>>378510
Ты же сам спросил - идти на хуй или нет. И не говори матеша. Просто не говори так. Ты же в мат треде, а не в б. Здесь многие с публикациями.
>>378533>>378539
#377 #378533
>>378532

>Здесь многие с публикациями.


УУУУ, прямо в ежегодных сборникх статей Усть-Пиздюйского училища математики? Ого прямо.
>>378534
#378 #378534
>>378533
А ты думаешь даже в вестнике усть пиздюйского политеха кого угодно публикуют что ли. А вообще ты не допонял, никакой агрессии, тут все твои друзья.
>>378535
#379 #378535
>>378534

>А ты думаешь даже в вестнике усть пиздюйского политеха кого угодно публикуют что ли


Кончено. Знаком с такой хуйней в универе. На конференции ходят со всякой хуитой (не все офк) ради бесплатной публикации для магистратуры, например.

>никакой агрессии, тут все твои друзья


Я знаю.
120 Кб, 1280x898
#380 #378536
>>378478
Отображение π:E→B проецирует все точки щетины на расчёске в их корень.

Прообраз точки на цилиндре будет одной щетинкой, волокном. У нас это называют слоем.
#381 #378537
>>378512
Ты гонишь. Мы на втором курсе по ней топологию учили.
>>378552
#382 #378538
>>378536
Ага, хуевый перевод. Для Hopf fibration выбрать перевод как расслоение Хопфа - пиздец. Нормальные люди перевели бы как фибрация Хопфа, но нет.
>>378540
#383 #378539
>>378532
Иди нахуй, школьник. Вот по такому школьному пафосу можно гарантированно детектировать школьников либо особо туповатых первокурсников.
>>378544
#384 #378540
>>378538
А с чего ты взял, что это вообще перевод, и тем более что это перевод с английского, а не французского?
>>378542
#385 #378541
>>378510
Хатчер норм. Но зависит от того, что ты знаешь вообще в математике. А что ты знаешь, кстати?

>топология интересна или мне идти на хуй?


Хуй ответишь на этот вопрос. Кто-то скажет, что на двачах неинтересно сидеть. Кто-то скажет, что это единственное, что интересно в жизни. И как теперь отвечать на

>каково сидеть на двачах, интересно или мне идти на хуй?

#386 #378542
>>378540
Открыл-то Хопф. Вообще похуй, какой язык в оригинале. Тут же проблема как с переводом термина quantum entanglement в квантовой механике - переводят его как квантовая запутанность, хотя лучше передает смысл перевод как квантовая сцепленность.

>А что ты знаешь, кстати?


А нихуя. Школьный курс + пара семестров высшей математики уровня забытых интегралов и ангема. Заценю ту книженцию, что челик выше кинул, сам себе отвечу.

>каково сидеть на двачах, интересно или мне идти на хуй?


Интересно, дохуя разных людей, хорошо что есть тематика.
>>378545
#387 #378544
>>378539
Подождите минуточку, я же только что на хуй кого-то послал, и тут же меня послали, даже не выдержав паузу? Это подрывает авторитет обсуждений.
#388 #378545
>>378542
Кстати а че вы так к школьникам отрицательно относитесь. Веди сами же практически школьники. Хули тебе три-джва года универа ума прибавит что ли. Все равно мамке трусы стирает и картофан жарит. Весь б забит магами которые ноют что им только вчера было 18. Возраст это полная хуета. Я сформировался лет в 12. И дальше мне похуй, автопилот
#389 #378546
>>378545

>мамке трусы стирает


мамкА а не мамке, быстрофикс ебта
>>378547
#390 #378547
>>378545
>>378546

>мамке трусы стирает


По Фрейду.
>>378548
#391 #378548
>>378547
у фрейда немецкий, как и русский, с падежами ебаными. а в английском таких оговорок невозможно.
#392 #378550
>>378545
Да нормально мы к школьникам относимся)
#393 #378552
>>378537

>Мы на втором курсе


В ВШЭ? Все равно не оч верю.
#394 #378555
>>378478
>>378479
Так что решили насчет канонiчного перевода fiber?
>>378556
65 Кб, 805x1218
87 Кб, 739x636
#395 #378556
>>378555
Слой.
>>378563
#396 #378557
>>378536
Блин, так теперь же всё понятно. А я-то голову ломал, почему так называется.
#397 #378558
>>378536
Сначала подумал, что это пост шизика или перевод промптом, а потом понял глубинный смысл.
35 Кб, 742x612
#398 #378563
>>378556
Математики - пидорасы.
>>378564
#399 #378564
>>378563
Что сказать-то хотел?
>>378571
#400 #378568
Вот же поехавшие. Это слово на русский перевел Александров, а он работал вместе с этим самым Хопфом и написал книгу в соавторстве. Очевидно, что русский термин самый подходящий. И вообще не понимаю нахуя вы переводите этот термин с английского, если он немецкого происхождения.
>>378571
12 Кб, 665x194
#401 #378571
>>378564
Что хотел то сказал.
>>378568
Нормальные люди инженеры и физики нормально все перевели. Хуле там не слоеная оптика, а?
>>378573>>378577
1295 Кб, 1210x1364
#402 #378573
>>378571
долбоебы fiber это клетчатка. у меня дедушка пикрил употребляет, чтобы газы не мучали.
#403 #378577
#404 #378580
Почему средь математиков столько безумцев? Вот случайно сейчас наткнулся на блог сумасшедшего, он у мамы и антифа, и математишн, и учебник свой пишет, но в основном про проституток. Вы все такие, да?
Алсо, классная шапка, выглядит внушительно
#405 #378581
>>378505
Математика это раздел физики, это общеизвестно же.
>>378632>>378638
#406 #378582
Подскажите книжек по топологии, если

>О. Я. Виро, О. А. Иванов, Н. Ю. Нецветаев, В. М. Харламов: “Элементарная топология”.


Совсем не идет.
>>378585
#407 #378583
>>378580

>Вы все такие, да


Лично я - да. Просто мы интересные люди, лол.
#408 #378585
>>378582
Брат по несчастью. Тоже с этого начал (после Куранта загорелся) и был дико удивлён такой скучной аксиоматикой и разжёвыванием.
Прасолов с его наглядной топологией нормас. Первые задачи интересно в уме решать, прокачиваешь пространственное мышление. Пятая глава о топологии в курантовской "Что такое математика?" - то, с чего я начал и тебе рекомендую. И наглядная топология от Болтянского хорошо заходит, особенно мне, ибо про графы я и так знаю и главы про них пролистал. Если хочешь прям с аксиом - на лекториуме есть курс лекций.
#409 #378587
>>378580
Это который хекслер? Ты Вербицкого погугли, вообще охуеешь.
>>378619
#410 #378588
Хватает ли математического материала на русском языке для изучения математики до, допустим, современного исследовательского уровня? Например, теории чисел.
>>378590
#411 #378590
>>378588
Нет. Русского материала хватит только чтобы докачаться до уровня семидесятых. Дальше придётся читать научные статьи, а они на английском.
>>378591
#412 #378591
>>378590
Ты дурной?!
Тот же Кострикин 2004 года, например.
>>378592
#413 #378592
>>378591
Я не дурной.
>>378593
#414 #378593
>>378592
А чего такие глупости говоришь?
>>378595
#415 #378595
>>378593
Я не говорю глупости.
>>378596>>378598
#416 #378596
>>378595
Докажи.
#417 #378598
>>378595

>Я не говорю глупости.


>Русского материала хватит только чтобы докачаться до уровня семидесятых


>семидесятых

>>378599>>378600
#418 #378599
>>378598
Угу.
#419 #378600
>>378598
Разве всё не так? Именно до семидесятых, потому что дальше идут статьи. А статьи на ангельском.
#420 #378603
>>378580

> он у мамы и антифа


> и математишн


> и учебник свой пишет,


> но в основном про проституток


Не вижу ничего безумного в вышеперечисленном.

Вообще, мне кажется, математикой занимаются люди, которых мало заботит мнение общества о них, потому что это не самая престижная профессия при капитализме. Если человек независим от мнения общества и у него творческий склад ума, то, скорее всего, у него будут какие-то необычные интересы и необычное поведение, вот и все. Но безумие тут ни при чем.
#421 #378619
>>378587
ну ты сравнил жопу с пальцем. вербицкий это голова и ваще. не то что этот вниманиеблядь дрищ.
#422 #378627
Эй, гомотопозависимые, я правильно понимаю что есть две математики, одна для технарей, с обоснованиями на примерах и без залезания в залупу, и вторая для математиков, с обонованиями в абстракциях и раздрачиванием простейших определений!?
>>378629
#423 #378629
>>378627
Нет, математика одна, просто для технарей упрощённая.
>>378633
#424 #378632
>>378581
Ты ахуел? Математика - отдельная хуйня, она, как мне кажется, от физики сильно отличается. Без математики не было бы физики, без физики - математика возможна.
Алсо, математика очень странная штука, ну, в совокупности с физикой - то есть всякие уравнения и равенства. Она описывает падающее яблоко, но ведь природа не решает никаких уравнений, яблоко просто падает.
>>378645
#425 #378633
>>378629
Ну да, как же.
То то в школах определяют натуральные числа
>>378634
#426 #378634
>>378633
Для школьников ещё проще, чем для технарей, очевидно. Что не так-то?
>>378637
#427 #378637
>>378634
Все так.
Как я и говорил, обычным людям доказательства на примерах и образах, для математиков доказательства с полным отрывом от реальности и удвоения объема шара разрезанием.
>>378642>>378644
#428 #378638
>>378581

>Математика это раздел физики


в нашем треде за такое убивают нахер
#429 #378642
>>378637
Как я и говорил, математика везде одна, только технарям и школьникам дают урезанную версию, первым для вычислений, а вторым просто так. И не надо думать, что студентом-математикам не дают хороших примеров и не объясняют "на пальцах".
>>378648
#430 #378644
>>378637
Алсо, если бы была цель технарям или школоте объяснить все те же доказательства, что и студентам-математикам, изложение тоже было бы запутанное, от этого никуда не денешься. А удвоение шара разрезанием внезапно считается контринтуитивным среди математиков, они же не шизофреники поголовно, блять. Некоторые из них даже стараются не пользоваться аксиомой выбора, чтобы не было таких парадоксов.
покормил
>>378648
#431 #378645
>>378632
Живые существа миллиарды лет эволюционировали, чтобы развивать фантазию - способность моделировать окружающий мир. Кто хорошо моделировал - много жрал. Кто плохо моделировал - того сжирали.

По-видимому, человек - одно из умнейших существ, созданных эволюцией. Человеческая способность моделировать мир в своем воображении очень высока. Люди -- хорошие фантазёры. В человеческом воображении есть очень много фантастических образов - звезда, человек, яблоко, человек Петя Иванов, непосредственно наблюдаемое мною прямо сейчас яблоко, etc. Важно понимать, что никто никогда не видел звезду; звезда - это фантастическая конструкция.

Математика - это деятельность человека по оттачиванию способности моделировать. Математики профессионально работают с фантастическими объектами, созданными человеческой фантазией. Они подвергают эти объекты многократному разложению и синтезу, смешивая их в причудливых сочетаниях. Математики - это профессиональные фантазёры.

Поэтому неудивительно, что математика иногда пригодна для описания реальности. Всё дело в миллиардах лет планетарной эволюции и капельке рефлексии.
>>378646>>378647
18 Кб, 574x576
#432 #378646
#433 #378647
>>378645

>звезда - это фантастическая конструкция


Че ты говоришь такое, на улице давно был? В окно посмотри, там солнце на небе. Солнце - это звезда.

>Математики - это профессиональные фантазёры.


Да ну, это слишком расплывчато сказано.
Я бы сказал, что математика - инструмент для предсказаний (моделирования, как ты сказал), но есть одна хуйня - схуя ли математика вообще работает, и хуйня еще удивительнее - схуя ли с её помощью можно предсказывать-моделировать? То есть, грубо говоря, почему сначала математик пишет систему уравнений, а потом физик находит в природе явление/объект, который и был предсказан той системой. Это же пиздец, природа не решает уравнений, ты только вдумайся в курсив, и в то, что схуя ли математика работает, она ж никому ничего не должна.
>>378649>>378670
#434 #378648
>>378642
>>378644
Ага, как же. Тот же парадокс про шар появился из за того что теория множеств напрочь оторванна от реальности.
И математиков так же учат, в полном отрыве от реальности, чтобы они тоже оторвались от реальности и начали на многообразия дрочить и обмазываться гомотопическим хаосом.
>>378650
#435 #378649
>>378647
Потомк что матемтика это образ физики, которая в свою очередь образ наблюдаемой вселенной.
>>378651>>378678
#436 #378650
>>378648

>Ага, как же


Хуяк же. По делу есть что сказать?
>>378654
#437 #378651
>>378649
Что такое образ? Типа бледная копия? Даже если так, из того, что ты написал, не следует, что математика со своими уравнениями должна работать во вселенной.
#438 #378654
>>378650
Возьми книжку по математике для математиков и для технарей и сравни их.
Если бы у одних была урезанная версия а у вторых полная, в первой книжке было бы написанно больше текста со следствиями и причинами.
Но на деле получается что в книжкх для математиков все написанно мутно и непонятно.
>>378656>>378661
#439 #378656
>>378654
Во-первых, кому непонятно-то? Мне понятно. А во-вторых, книжка по математике "для технарей" -
это сборник руководств по решению задач тащемта. Откуда эти руководства берутся, по-твоему? Из какой-то другой математики или из воздуха?
>>378658
#440 #378658
>>378656
Зайдем с другой стороны.
Дай определение дифференциала с точки зрения математики для математиков.
>>378671
#441 #378661
>>378654

>все написанно мутно и непонятно


Это называется абстрактно.
#442 #378663
Мне кажется еще проблема в том, что всякие пидоры на ютюбе толкают разный научпоп и генерят миллионы просмотров своего говна. Вот что может подумать нуб посмотрев такую блевотину например:

https://www.youtube.com/watch?v=w-I6XTVZXww

То, что это makes no fucking sense следовательно матиматика это хуита.

4 МИЛЛИОНА просмотров блять.
>>378667>>378673
#443 #378665
Пытаюсь вспомнить, какой учебник мат. статистики мне понравился два месяца назад, но я забыл его название. Помогите найти этот учебник. Это не Гмурман, не Кремер и не задачник Свешникова.
#444 #378667
>>378663
Научпоп - зло, ебать-копать.
>>378672
#445 #378670
>>378647
Есть объективная реальность, существующая независимо от твоего мышления, а есть образы, существующие в твоей голове. Когда на твои органы чувств действует некоторое ощущение, в твоей голове возникает восприятие. Когда ты фантазируешь, в твоей голове возникает представление (термины позаимствованы из учебника логики Гетмановой).

Восприятие - сложная вещь. Если на один и тот же луг выйдут генерал, крестьянин, художник и травник, то они воспримут его по-разному. Генерал прикинет, сколько можно нарыть окопов. Крестьянин оценит, сколько можно посадить картошки. Художник восхитится красотой. Травник присмотрится к траве, которая растёт на этом лугу. Одни и те же ощущения вызовут у этих людей разные восприятия.

Восприятие индивидуально, в нем участвует не только то, что непосредственно действует на органы чувств, но вообще весь опыт человека. У разных людей опыты разные, поэтому восприятия у них тоже разные. Фантазия о звезде, которая есть у меня, сильно отличается от фантазий о звездах, которые были у моих предков тысячи лет назад. Я воспринимаю звезду как шарообразный сгусток плазмы. Мои предки воспринимали звезду как дырку в твердом куполе. Мои потомки, возможно, будут воспринимать звезду как решение некоторого уравнения.

Конечно, ввиду большого генетического сходства, люди воспринимают мир в основном одинаково. Мозг всех людей способен работать со временем и выделять в пространстве разнообразные геометрические формы. У всех людей есть представление о стабильном объекте (оно, кстати, не врожденное, а формируется на первом году жизни). Почти у всех людей есть отделы мозга, отвечающие за речь и арифметику. Все люди оборудованы памятью и некоторой фантазией. Кроме того, все люди, живущие в достаточно цивилизованном государстве, имеют один и тот же набор книг, смотрят один и тот же телевизор, упарываются одной и той же фантастикой. Поэтому у всех людей, проживающих в одно и то же время на одной и той же территории, фундаментальные фантазии, типа как о звезде, похожи. Но фантазия о звезде - это всё-таки фантазия.

Когда я смотрю на Солнце, я ощущаю ощущения, а не звезду. Звезда возникает в моей фантазии на основе всего моего жизненного опыта и некоторых аппаратных особенностей моего мозга.
#445 #378670
>>378647
Есть объективная реальность, существующая независимо от твоего мышления, а есть образы, существующие в твоей голове. Когда на твои органы чувств действует некоторое ощущение, в твоей голове возникает восприятие. Когда ты фантазируешь, в твоей голове возникает представление (термины позаимствованы из учебника логики Гетмановой).

Восприятие - сложная вещь. Если на один и тот же луг выйдут генерал, крестьянин, художник и травник, то они воспримут его по-разному. Генерал прикинет, сколько можно нарыть окопов. Крестьянин оценит, сколько можно посадить картошки. Художник восхитится красотой. Травник присмотрится к траве, которая растёт на этом лугу. Одни и те же ощущения вызовут у этих людей разные восприятия.

Восприятие индивидуально, в нем участвует не только то, что непосредственно действует на органы чувств, но вообще весь опыт человека. У разных людей опыты разные, поэтому восприятия у них тоже разные. Фантазия о звезде, которая есть у меня, сильно отличается от фантазий о звездах, которые были у моих предков тысячи лет назад. Я воспринимаю звезду как шарообразный сгусток плазмы. Мои предки воспринимали звезду как дырку в твердом куполе. Мои потомки, возможно, будут воспринимать звезду как решение некоторого уравнения.

Конечно, ввиду большого генетического сходства, люди воспринимают мир в основном одинаково. Мозг всех людей способен работать со временем и выделять в пространстве разнообразные геометрические формы. У всех людей есть представление о стабильном объекте (оно, кстати, не врожденное, а формируется на первом году жизни). Почти у всех людей есть отделы мозга, отвечающие за речь и арифметику. Все люди оборудованы памятью и некоторой фантазией. Кроме того, все люди, живущие в достаточно цивилизованном государстве, имеют один и тот же набор книг, смотрят один и тот же телевизор, упарываются одной и той же фантастикой. Поэтому у всех людей, проживающих в одно и то же время на одной и той же территории, фундаментальные фантазии, типа как о звезде, похожи. Но фантазия о звезде - это всё-таки фантазия.

Когда я смотрю на Солнце, я ощущаю ощущения, а не звезду. Звезда возникает в моей фантазии на основе всего моего жизненного опыта и некоторых аппаратных особенностей моего мозга.
#446 #378671
>>378658
Дифференциала чего? Их как бы много разных.
>>378711
#447 #378672
>>378667
Но дело не только в научнпопе, а в том что они через научпоп каналы сеят мракобесие - "присваивание" результата расходящимся рядам это только в дурке можно додуматься делать. Всякие гранди-хуянди чезаро-хуяро это полная абстракная чушь, сектарианство, не имеющее ничего общего с объективной реальностью.
>>378676
#448 #378673
>>378663
Ты что-то против Рамануджана, сука, имеешь?
>>378675
#449 #378675
>>378673
А интересно, это в универе проходят? Мне бы сука СТЫДНО было бы преподу такую хуету сдавать, и гнать не краснея что сумма 1-1+1-1... равна 1/2 потому что ебучий чезаро так решил. Это же унижение, как в джинсиках с подворотом ходить и с косичкой.
>>378682
#450 #378676
>>378672
Это не мракобесие, это легально. Но в этом-то и проблема, что несведущий человек толком и не поймёт, почему это так и как это вообще нужно понимать, и сделает пародоксальный вывод о том, что это не он не понял, а ему хуйни наплели.
>>378678
#451 #378678
>>378676
Define понимание. Можно "понять" - просто выучить ПРАВИЛА и применять их. И что такое "легально"? Просто кто-то придумал некие правила суммирования. Можно ведь придумать как разделить на ноль и назвать эту теорию своим именем. Вот это утверждение как согласуется с фокусами с рядами? >>378649
>>378681
#452 #378679
Разумно ли, о мудрейшиеъ, восполнять пробелы в школьных знаниях, по учебникам А П Киселеваъ, о котором столько восторженныхъ отзывов в сетиъ?
#453 #378681
>>378678
У слова понять один и только один смысл
>>378685
#454 #378682
>>378675

>Мама, я траллю двач!

>>378685
#455 #378685
>>378681
Ну вот бывает так, что ты чего-то не понимаешь и думаешь что это хуета. И считает наверное это потому что ты не обладаешь ЗНАНИЯМИ. Тогда ты изучаешь вопрос, веришь выкладкам и у тебя наступает ОЗАРЕНИЕ. Наконец то я понял! Но через некоторое время когда все отложится, ВНЕЗАПНО осознаешь, что нет, все таки хуета это. Зато теперь ты знаешь почему это хуета. Потому что ты понял ход мысли автора, но не купился. Как то так.

>>378682
Почему тралю, меня реально раздражают несходящиеся ряды и вольное обращение с ними.
>>378687>>378688
#456 #378687
>>378685
Какой раздражительный. Это вольное обращение придумали люди поумнее тебя.
#457 #378688
>>378685
Что ещё за "вольное обращение"? Просто ввели обозначение такое, не хуже других
#458 #378692

>манга гайд ту...


>без фансервиса


Безполезно.
39 Кб, 852x280
#459 #378695
Почему там такая матрица? Должна быть же
0 0 1
1 0 0
0 1 0
>>378699
36 Кб, 685x630
#460 #378697
Сап, тредчане.
Годная книжка? Стоит ли тратить на неё время?
#461 #378699
>>378695
Там опечатка, очевидно.
>>378702
#462 #378702
>>378699
Ну вдруг я не понял чаго.
#463 #378711
>>378671
Функции, очевидно же что не автомобильный.
>>378713
#464 #378713
>>378711
Пусть V и W - нормированные векторные пространства, полные по метрике, порождённой нормой. Нормы обозначим как nV и nW.

Пусть U - открытое подмножество V. Пусть f - функция из U в W. Пусть x - точка U.

Ограниченная линейная функция L из V в W называется дифференциалом Фреше функции f в точке x, если предел отношения nW(f(x+h) - f(h) - L(h)) / nV(h) равен нулю при h стремящемся к нулю по множеству x+h ∈ U.

Упрощая, дифференциал - это линейный оператор, приближающий функцию с малой погрешностью.

Если пространства V и W конечномерны и в них выбраны базисы, то всякое линейное отображение из V в W биективно соответствует некоторой матрице. Матрица дифференциала Фреше называется производной Фреше. В случае когда V=W=R эта матрица отождествляется с числом, которое и оказывается, очевидно, обычной производной функции в точке.
>>378768>>378792
#465 #378725
>>377293 (OP)
Реквестирую книгу по теории колец главных идеалов
>>378762
304 Кб, 1000x1500
#466 #378734
Сколько всего разных строчек можно сделать с помощью картинки?
Как считают подобное?
>>378735>>378736
70 Кб, 604x404
#467 #378735
>>378734
Бля, перепутал пикчи
#468 #378736
>>378734
Всего 10 столбцов, каждый из 10 строк. Первая строчка выбирается из первого столбца, т. е. есть 10 различных первых строчек. Для каждой первой строчки можно выбрать одну из возможных вторых строчек, их тоже десять. Стало быть, комбинаций первых и вторых строк всего 10*10. Таким образом, получается, что вариантов стиха из 10 строк будет 10^10.
>>378737>>378738
#469 #378737
>>378736
А можно таким макаром себе речь на выступление сделать?
#470 #378738
>>378736
Десять миллиардов стихов. Найс.
>>378739
#471 #378739
>>378738
Больше, чем люди написали за всю свою историю. В одной какой-то дурацкой картинке 300х400. На Дваче.
10 Кб, 651x178
#472 #378747
#473 #378749
Анон, объясни максимально простым языком, как рисовать множество Мандельброта. Я идиот, потому что прогуливал школу, совсем ничего не могу понять в найденных мной мануалах.
>>378753
#474 #378750

>Я идиот, потому что прогуливал школу


>рисовать множество Мандельброта


В школе его не проходят.
>>378751
#475 #378751
>>378750

>В школе его не проходят.


ПТУ я тоже прогуливал.
>>378752
#476 #378752
>>378751
И в пту тоже. Даже в университе наврядли, только если в спец-курсе каком-нибудь.
#478 #378754
>>378753
Естественно я прочитал это. Но я настолько туп, что не понял даже вот этот мануал.
http://www.wikihow.com/Plot-the-Mandelbrot-Set-By-Hand
>>378755>>378756
#481 #378762
>>378725
Бамп
#482 #378768
>>378713
А теперь скажи чем это определение отличается от того которое изобрел ньютон с лейбницем.
>>378769>>378785
#483 #378769
>>378768
Ньютон с Лейбницем не знали о множествах, метрических пространствах и нормах. У них он для R, а в определении обобщении на большее количество пространств.
>>378770
#484 #378770
>>378769
Ты не к словам придирайся, а к сути.
>>378771
#485 #378771
>>378770

>Ты не к словам придирайся, а к сути.

>>378772
#486 #378772
>>378771
Ты тролируешь двач так?
Суть дифференциала поменялась или нет?
>>378773>>378775
#487 #378773
>>378772

>А теперь скажи чем это определение отличается от того которое изобрел ньютон с лейбницем.



>Ты не к словам придирайся, а к сути.



>Ты тролируешь двач так?


>Суть дифференциала поменялась или нет?


Ты сначала с вопросом определись.
#488 #378775
>>378772
Тебе ещё блять растолковать надо, существуют полные метрические пространства помимо R? Или что общие определения и теоремы лучше частных, потому что они могут быть более широко применимы?
>>378784
#489 #378784
>>378775
Ну давай, примени мне производную на какой то, любой, случай.
#490 #378785
>>378768
Начнём с главного. Ни Ньютон, ни Лейбниц не использовали дифференциал в смысле "линейное отображение". Такой смысл начал вкладывать в этот термин лишь Коши.
>>378786>>378787
#491 #378786
>>378785
Так, давай с самого начала появления фиддеренциала

>Производная (функции в точке) — основное понятие дифференциального исчисления, характеризующее скорость изменения функции (в данной точке)


Вот теперь из этой точки показывай дорожку к нормированным векторным пространствам.
>>378787>>378791
#492 #378787
>>378786
>>378785
Уточню, я не прошу чтобы ты мне сказал "вот пал палыч палкин сказал это", а показал где, в каком месте пал палычу палкину пришлось столкнуться с тем чтобы прийти к векторным пространствам.
>>378788
#493 #378788
>>378787
Уже просто R - векторное пространство.

1. Изучал функции Rm->Rn
2. Перешёл к функциям Cm->Cn
3. Решил не ограничиваться конечномерным случаем
#494 #378791
>>378786
1. Линейные функции из вещественных чисел в вещественные числа - это всегда умножение на какое-то число. Обратно, о каждом числе можно думать как о линейной функции. Более того, линейная функция из R^m в R^n биективно соответствует матрице размера m на n.

2. Функции из R в R мы будем называть числовыми функциями одного аргумента. Функции из R^m в R мы будем называть числовыми функциями нескольких аргументов. Функции из R^m в R^n мы будем называть векторными функциями нескольких аргументов.

3. Известно, как определить производную числовой функции одного аргумента. Производная таких функций - это число. Умножение на это число - по пункту 1 линейная функция. Она называется дифференциал в точке. У дифференциала есть главное свойство: дифференциал приближает функцию с малой погрешностью. Это свойство следует из определения производной.

4. Для векторных функций нескольких аргументов тоже бы хотелось иметь дифференциал. Для этого нужно
а) указать линейную функцию
б) уточнить понятие "приближать с малой погрешностью"

5. Рассмотрим сначала числовые функции нескольких аргументов. Известно, что для таких функций введено понятие "частной производной" в точке.

6. Заметим, что векторную функцию нескольких аргументов можно рассмотреть как несколько скалярных функций (от тех же самых аргументов). То есть перейти к координатам. Например, функцию в пространство можно рассмотреть как три числовые функции - координаты по осям OX, OY, OZ. Функцию из R^m в R^n можно рассмотреть как n функций из R^m в R.

7. Возьмём векторную функцию нескольких аргументов. Рассмотрим её как несколько скалярных функций. У каждой из скалярных функций возьмём все частные производные. Составим из них матрицу. Так как всякая матрица однозначно соответствует линейному оператору, тем самым получим линейный оператор. Он называется якобиан.

8. Зафиксируем некую точку, назовём её a. Пусть у нас есть какая-то функция. В точке a у неё одно значение. Если мы немного сдвинемся от точки a, у функции станет другое значение.

9. Пусть у нас есть две функции, f и L. Скажем: функция L приближает функцию f в точке a, если для любого маленького сдвига от точки a расстояние между изменившимися значениями f и L мало по сравнению со сдвигом. Что значит мало? Сдвиг - это вектор. Расстояние между двумя точками - тоже вектор. Под малостью мы будем понимать малость длин этих векторов, или малость норм. Т.е. просто малость чисел.

10. Малость чисел мы будем понимать в нотации o малое. Смысл предыдущих пунктов, стало быть, таков. Функция L приближает функцию f в точке a, если существует такая числовая функция r, что
а) r бесконечно-малая
б) |f(a+x)-f(a) - L(a+x)-L(a)| = |x|r(x).
Под x понимается сдвиг от точки a.
f(a+x)-f(a) - это приращение функции f при сдвиге x от точки a.
L(x+a)-L(a) - это приращение функции L при сдвиге x от точки a.
Если функция L линейна, то L(x+a)-L(a) = L(x).
В дальнейшем будем предполагать, что L линейна.

11.Разделив обе части на |x| и перейдя к пределу x->0, получим выражение, которое очень напоминает аналогичное выражение для производной числовых функций.
lim |f(a+x)-f(a) - L(x)|/|x| = 0.

12. В п. 7 мы получили линейный оператор, якобиан. Если он приближает функцию в смысле п. 11, то якобиан называется дифференциал.
#494 #378791
>>378786
1. Линейные функции из вещественных чисел в вещественные числа - это всегда умножение на какое-то число. Обратно, о каждом числе можно думать как о линейной функции. Более того, линейная функция из R^m в R^n биективно соответствует матрице размера m на n.

2. Функции из R в R мы будем называть числовыми функциями одного аргумента. Функции из R^m в R мы будем называть числовыми функциями нескольких аргументов. Функции из R^m в R^n мы будем называть векторными функциями нескольких аргументов.

3. Известно, как определить производную числовой функции одного аргумента. Производная таких функций - это число. Умножение на это число - по пункту 1 линейная функция. Она называется дифференциал в точке. У дифференциала есть главное свойство: дифференциал приближает функцию с малой погрешностью. Это свойство следует из определения производной.

4. Для векторных функций нескольких аргументов тоже бы хотелось иметь дифференциал. Для этого нужно
а) указать линейную функцию
б) уточнить понятие "приближать с малой погрешностью"

5. Рассмотрим сначала числовые функции нескольких аргументов. Известно, что для таких функций введено понятие "частной производной" в точке.

6. Заметим, что векторную функцию нескольких аргументов можно рассмотреть как несколько скалярных функций (от тех же самых аргументов). То есть перейти к координатам. Например, функцию в пространство можно рассмотреть как три числовые функции - координаты по осям OX, OY, OZ. Функцию из R^m в R^n можно рассмотреть как n функций из R^m в R.

7. Возьмём векторную функцию нескольких аргументов. Рассмотрим её как несколько скалярных функций. У каждой из скалярных функций возьмём все частные производные. Составим из них матрицу. Так как всякая матрица однозначно соответствует линейному оператору, тем самым получим линейный оператор. Он называется якобиан.

8. Зафиксируем некую точку, назовём её a. Пусть у нас есть какая-то функция. В точке a у неё одно значение. Если мы немного сдвинемся от точки a, у функции станет другое значение.

9. Пусть у нас есть две функции, f и L. Скажем: функция L приближает функцию f в точке a, если для любого маленького сдвига от точки a расстояние между изменившимися значениями f и L мало по сравнению со сдвигом. Что значит мало? Сдвиг - это вектор. Расстояние между двумя точками - тоже вектор. Под малостью мы будем понимать малость длин этих векторов, или малость норм. Т.е. просто малость чисел.

10. Малость чисел мы будем понимать в нотации o малое. Смысл предыдущих пунктов, стало быть, таков. Функция L приближает функцию f в точке a, если существует такая числовая функция r, что
а) r бесконечно-малая
б) |f(a+x)-f(a) - L(a+x)-L(a)| = |x|r(x).
Под x понимается сдвиг от точки a.
f(a+x)-f(a) - это приращение функции f при сдвиге x от точки a.
L(x+a)-L(a) - это приращение функции L при сдвиге x от точки a.
Если функция L линейна, то L(x+a)-L(a) = L(x).
В дальнейшем будем предполагать, что L линейна.

11.Разделив обе части на |x| и перейдя к пределу x->0, получим выражение, которое очень напоминает аналогичное выражение для производной числовых функций.
lim |f(a+x)-f(a) - L(x)|/|x| = 0.

12. В п. 7 мы получили линейный оператор, якобиан. Если он приближает функцию в смысле п. 11, то якобиан называется дифференциал.
>>378801>>378833
#495 #378792
>>378713

>f(x+h) - f(h)


Должно быть f(x+h) - f(x) , конечно. Опечатка.
#496 #378794
Анон, как доказать, что числой в пятой степени оканчивается на ту же степень, что и число в первой?

не ругайтесь, я мимошкольник
>>378796>>378797
#497 #378796
>>378794
Для 1-9 непосредственной проверкой. Многозначное число представлять как сумму числа, оканчивающегося на 0, и однозначного числа.
#498 #378797
>>378794
phi(10)=4, поэтому по теореме Эйлера в Z/10Z a^5=a. ЧТД.
93 Кб, 667x463
#499 #378801
#500 #378808
Перекот

>>378807 (OP)
#501 #378833
>>378791

>Пусть у нас есть две функции, f и L. Скажем: функция L приближает функцию f в точке a, если для любого маленького сдвига от точки a расстояние между изменившимися значениями f и L мало по сравнению со сдвигом. Что значит мало? Сдвиг - это вектор. Расстояние между двумя точками - тоже вектор. Под малостью мы будем понимать малость длин этих векторов, или малость норм. Т.е. просто малость чисел.



Ты только доказываешь

>У дифференциала есть главное свойство: дифференциал приближает функцию с малой погрешностью


как на него не смотри.
Тред утонул или удален.
Это копия, сохраненная 16 ноября 2016 года.

Скачать тред: только с превью, с превью и прикрепленными файлами.
Второй вариант может долго скачиваться. Файлы будут только в живых или недавно утонувших тредах. Подробнее

Если вам полезен архив М.Двача, пожертвуйте на оплату сервера.
« /sci/В начало тредаВеб-версияНастройки
/a//b//mu//s//vg/Все доски